PA Easy Family Medicine

¡Supera tus tareas y exámenes ahora con Quizwiz!

A 60-year-old female presents to her primary care practitioner for her yearly check up. Her past medical history is significant only for a history of repeat kidney stones. She enjoys gardening outside, has no significant family history, and is not taking any medications. Routine mammograms have been normal. During the review of her systems, the patient describes feeling tired lately and has noticed muscle aches over the last few weeks. Initial lab values are shown below: Hemoglobin= 12.0 gm/dL Hematocrit = 36% BUN= 18 mg/dL Creatinine= 1.0 mg/dL Calcium= 12 mg/dL Phosphate= 2.0 mg/dL Intact PTH= 80 pg/mL TSH= 3.0 uU/L What is the most likely pathology associated with the findings in this patient? A Solitary parathyroid adenoma B Excessive Ingestion of calcium C Parathyroid carcinoma D Medullary thyroid cancer

A The correct choice is A, solitary parathyroid adenoma. Primary hyperparathyroidism is caused by a solitary adenoma in 80% of patient cases. Choices B, D, and E do not cause primary hyperparathyroidism. Choice C can cause primary hyperparathyroidism, but with an incidence of less than 1%.

A 78-year-old male returns to the FP office for a follow up of non-insulin-dependent diabetes mellitus (NIDDM) as a new patient to you, although he has been an office patient for the past year. He denies any problems this visit and says his blood sugars are in the 90-120 mg/dl range. He is currently taking the medications listed in the following choices. You receive his labs and note that his creatinine is 2.0 mg/dl and on the previous few labs this creatinine was also in the 1.8-2.0 mg/dl range. What medication should be discontinued? A glipizide B metformin C omeprazole D sitagliptin E atenolol

B The correct answer is (B). Metformin is contraindicated in this diabetic patient with chronic renal failure due to an increased risk of lactic acidosis and should be discontinued. Sitagliptin requires a dosing adjustment in renal failure but is not contraindicated in this patient. Glipizide (sulfonylurea), omeprazole (a PPI for GERD), and atenolol (a beta blocker for hypertension) are not contraindicated in this patient. Beta blockers should be used with caution in diabetics due to the potential of masking symptoms of hypoglycemia, but are not contraindicated.

42-year-old female with history of severe rheumatoid arthritis requiring multiple medications follows up with her primary care physician assistant. She has been doing well on her current medication regimen but has been having some mild headaches. Her blood pressure today is 170/92 P = 82, T = 96.2˚F. She denies any previous history of hypertension. Which of the following medications that she is taking is most likely to be contributing to her hypertension? A methotrexate B cyclosporine C acetaminophen D hydrocodone E fexofenadine

B The correct answer is (B). Cyclosporine, an immunosuppressant, is known to cause hypertension as a potential side effect and should be considered as a potential secondary cause of secondary hypertension. Methotrexate, acetaminophen, and fexofenadine are not known to cause secondary hypertension. Hydrocodone may actually cause hypotension as a potential adverse reaction.

A 29-year-old woman presents in July to your office with symptoms of palpitations, sore neck, and excessive sweating, despite using her air conditioner all the time. No surgical or trauma history is noted. She is currently not taking any medications. Vitals include the following: BP = 124/68, pulse = 110 beats per minute, respirations = 18 per minute, and temperature = 101 o F orally. Upon exam, her thyroid is mildly enlarged without nodules, and severely tender. No local erythema or heat is noted. Which of the following lab results would you expect in this patient? A Serum total T4 level = 5.0 ug/dL B Serum TSH level = 0.25 uIU/mL C Sedimentation rate = 15 mm/hr D Free thyroxine index = 8.0 E Positive thyroid stimulating antibodies

B The correct choice is B, Serum TSH level=0.25 uIU/mL. The reference range for TSH is 0.34 to 4.25 uIU/mL, and therefore the level in this patient is low. This patient is presenting with signs and symptoms of hyperthyroidism, most likely due to subacute thyroiditis. The leaking of thyroid hormone into the circulation causes anterior pituitary suppression and reduced TSH secretion. Choice A is seen in patients with hypothyroidism. Choice C is within the reference range for woman. Since subacute thyroiditis is an acute inflammatory disorder, patients with this disorder will commonly present with an elevated sedimentation rate. Choice D corresponds to a euthyroid situation. It is an estimate of the free thyroid hormone level in the plasma. This result is within the reference range. Choice E is not seen in subacute thyroiditis. They are commonly found in patients with Graves' disease.

A 36-year-old nurse states she will be traveling to Honduras for a medical mission and wants to ensure that she has adequate protection against hepatitis B. Which of the following laboratory values is consistent with immunity against hepatitis B from prior immunization? A +HBsAg, +anti-HBc IgM B +HBsAg, +anti-Hbc IgG C +anti-HBs D +anti-HBs, +anti-HBIgG E +anti-HCV

C The correct answer is (C). The presence of detectable anti-HBs indicates that she is immune from a previous hepatitis B vaccine. Booster immunizations are not indicated in her case. Choice (A) suggests acute hepatitis B. Choice (B) suggests she has highly infective chronic hepatitis B. Choice (D) suggests she had previous hepatitis B infection, recovered. Choice (E) suggests acute hepatitis C.

A patient presents with symptoms of palpitations, tremor, confusion, and sweating. She is also hungry. Her plasma glucose level is currently 54 mg/dL. Which of the following drugs would be particularly important to investigate during the patient history? A Vancomycin B Acetaminophen C Ethanol D Lisinopril E Prednisone

C The correct choice is C, ethanol. Ethanol can suppress gluconeogenesis and stimulate glucose utilization. Alcohol induced hypoglycemia occurs most commonly after a several day drinking binge with little food intake. Other medications that can be associated with hypoglycemia include insulin, insulin secretagogues, salicylates, sulfonamides, pentamidine, quinine, quinolone antibiotics, and beta 1 -adrenergic antagonists. The other choices are not associated with inducing hypoglycemia. Choice E, prednisone use, has been associated with hyperglycemia.

A 60-year-old female presents for a routine physical exam. She has no significant past medical history and is taking no medications. Her surgical history includes only carpal tunnel repair five years ago. Upon review of symptoms, she notes feeling somewhat tired. She attributes that to old age. Vital signs include BP 110/55, pulse 55 bpm, and respirations 16 per minute. Upon exam you note an enlarged, firm thyroid, thickening of her skin, and puffy facial features and pallor. A portion of blood tests that you ordered is shown below: Serum TSH = 4.9 uIU/mL Hemoglobin = 10.0 g/dL Hematocrit = 30% MCV = 101 Fasting plasma glucose = 105 mg/dL BUN = 10 mg/dL Creatinine = 0.6 ng/mL Which of the following is the most appropriate intervention? A Lithium carbonate 300 mg PO twice daily B Resection of the anterior pituitary C Levothyroxin 50 to 100 ug PO daily D Ferrous sulfate 325 mg PO three times daily

C The correct choice is C, levothyroxin 50 to 100 ug PO daily. This patient has classic signs and symptoms of hypothyroidism and required thyroid hormone supplementation. The most common form of hypothyroidism is primary hypothyroidism (e.g. Hashimoto's thyroiditis), and the most common thyroid hormone supplementation is levothyroxine.

Which of the following sets of disorders is commonly found in multiple endocrine neoplasia (MEN) 2A? A Islet cell tumor; renal cell carcinoma; pheochromocytoma; B Pheochromocytoma; medullary thyroid carcinoma; mucosal neuromas C Medullary thyroid carcinoma; parathyroid hyperplasia; pheochromocytoma D Parathyroid adenoma; islet cell hyperplasia; pituitary adenoma E Visceral lipomas; Marfanoid features; retinal angiomas

C The correct choice is C. The three primary features of MEN type 2A include medullary thyroid carcinoma, parathyroid hyperplasia or adenoma, and pheochromocytoma. In choice A, islet cell tumor and renal cell carcinoma are disorders found in various other MEN syndromes, but not in MEN type 2A. Choice B, pheochromocytoma, medullary thyroid carcinoma, and mucosal neuroma are disorders found in MEN 2B. Choice D, parathyroid adenoma, islet cell hyperplasia, and pituitary adenoma are found in MEN I. Choice E, visceral lipomas, Marfanoid features, and retinal angiomas are features found in various MEN syndromes, but not in MEN 2A.

A 23 year-old male with cystic fibrosis inquires about the availability of treatments that can help improve his lung function. Which of the following treatments is most effective at reversing the pulmonary effects of cystic fibrosis? A Albuterol B Azithromycin C Inhaled hypertonic saline D Inhaled levofloxacin E Ivacaftor

Correct Answer is: E Ivacaftor (E) is the only treatment that restores function of the CFTR protein in cystic fibrosis patients with a G551D mutation thereby reversing the effects of the disease, approximately 5% of all cystic fibrosis patients have the G551D mutation. Albuterol (A) and hypertonic saline (C) are indicated to improve lung function and mucous clearance. Azithromycin (B) and inhaled levofloxacin (D) are used to treat chronic infection/colonization with pseudomonous.

A 48 year-old male presents to the clinic with a history of productive cough for the past 3 days. Which of the following factors if present best predicts the diagnosis of pneumonia in this patient? A Adenopathy B Double sickening C Dyspnea D Fever E Hoarseness

D Cough, fever (D) and sputum production are classic symptoms of pneumonia. Adenopathy (A) and hoarseness (E) may coexist in some cases of viral pneumonia. Dyspnea (C) may develop as pneumonia progresses. Double sickening (B) is a clinical consideration in the differentiation of acute sinusitis from upper respiratory infection that typically occurs after approximately 7 days of illness.

A 34-year-old male presents to the primary care office with a complaint of heartburn. He has symptoms two to three times a week, and it occurs about 30 minutes after eating. He has tried over-the-counter antacids and they were helping to relieve his symptoms for a few months, but they are not working well now. He denies dysphagia, odynophagia, or weight loss. What is the next, most appropriate step in managing this patient? A Upper endoscopy B Increase the dose of the antacids C Barium esophagography D Treat empirically with a proton pump inhibitor E Esophageal pH monitoring

D The correct answer is to treat empirically with a proton pump inhibitor. This is the treatment of choice in a patient with typical symptoms of heartburn and regurgitation. If the patient fails therapy with a proton pump inhibitor, or had any alarm symptoms present, then it would be reasonable to start with an upper endoscopy, barium esophagography, or esophageal pH monitoring. Increasing the dose of the antacids is not appropriate management for this patient.

A patient was recently diagnosed with type 1 diabetes mellitus. A treatment plan was initiated, with a combination regimen of insulin. Which of the following types of insulin works well with a rapidly acting insulin, such as insulin lispro, to provide 24-hour coverage for the patient? A NPH insulin B Regular insulin C Insulin aspart D Insulin glargine E Humalog 75/25

D The correct choice is D, insulin glargine. This is the only long acting insulin listed. The combination of a long acting insulin with a rapidly acting insulin provides physiologic insulin replacement to the patient. This regimen provides postprandial control after meals and basal coverage throughout the day and night. Choice A NPH insulin, can be used by itself in two or more injections throughout the day. Choice B, regular insulin, can be used instead of rapid acting insulin, and not in combination with it. Choice C, insulin aspart, is a type of rapidly acting insulin and would not be used in combination with another rapidly acting insulin. Choice E, Humalog 75/25, is a combination insulin preparation with 75% intermediate acting insulin and 25% insulin lispro.

A female patient presents with a history of hypertension and low plasma HDL. She is asking you if she has metabolic syndrome. She does not have diabetes and she is not obese. Which of the following NCEP ATP III criteria would be needed to confirm that diagnosis? A LDL > 70 mg/dL B LDL > 90 mg/dL C Triglycerides > 100 mg/dL D Triglycerides ≥ 150 mg/dL E Triglycerides > 300 mg/dL

D The correct choice is D, triglycerides ≥ 150 mg/dL. The NCEP ATP III criteria for metabolic syndrome include three or more of the following: Central obesity with waist circumference > 102 cm in men and > 88 cm in women. Hypertriglyceridemia with serum triglycerides ≥ 150 mg/dL, or the patient is taking medication for hypertriglyceridemia. Low HDL level < 40 mg/dL in men and < 50 mg/dL in women, or the patient is taking medication for low HDL Hypertension with blood pressure ≥ 130 mm systolic or ≥ 85 mm diastolic, or the patient is taking medication for high blood pressure. Fasting plasma glucose ≥ 100 mg/dL, or a positive diagnosis for diabetes mellitus, or the patient is taking medication for hyperglycemia.

A combination of which of the following medications may increase the risk of muscle and liver disease more than either drug used alone? A Ezetimibe and HMG-CoA reductase inhibitor B Low dose niacin and HMG-CoA reductase inhibitor C Colestipol and gemfibrozil D Niacin and cholestyramine E Gemfibrozil and HMG-CoA reductase inhibitor

E The correct choice is E, gemfibrozil and HMG-CoA reductase inhibitor. In combination, these can potentiate the risk of developing hepatic disease or myopathy. Choice A, Ezetimibe and HMG-CoA reductase inhibitor, is a synergistic treatment plan for patients with primary hypercholesterolemia. Choice B, low dose niacin and HMG-CoA reductase inhibitor, is a practical and effective treatment plan for patients with familial combined hyperproteinemia. Choice C, colestipol and gemfibrozil, is sometimes useful in patients with familial combined hyperlipidemia who are intolerant of niacin or HMG-CoA reductase inhibitors. Unfortunately, this combination may increase the risk of cholelithiasis. Choice D, niacin and cholestyramine, is useful in disorders with elevated VLDL and LDL, and useful in treating heterozygous familial hypercholesterolemia.

A 35-year-old woman comes to see you with symptoms of anxiety, sweating, and tremors. She has no history of diabetes, liver or kidney failure, hormone deficiencies, or past surgeries. She states that she is not currently prescribed any medications and does not drink alcohol. She is concerned that she will not be able to continue to care for her husband, who has a long history of diabetes mellitus. She is asking you to give her some medication to stop her symptoms. Initial lab results are as follows: Plasma glucose = 54 mg/dL (70-110 mg/dL) TSH = 2.0 mIU/L (0.34-4.25 uIU/mL) Insulin = 35 uU/mL (2.0-20 uU/mL) C-peptide= 0.4 ng/mL (0.5-2.0 ng/mL) Her symptoms are relieved with the drinking of orange juice. What is the most likely cause of her hypoglycemia? A Alimentary hypoglycemia B Factitious hypoglycemia C Beta cell insulinoma D Congenital hyperinsulinism E Reactive hypoglycemia

Explanation B The correct choice is B, factitious hypoglycemia. This occurs when patients accidentally or on purpose self-administer insulin or an insulin secretagogue. This occurs most commonly among health care personnel, patients with diabetes or family members of those with diabetes, and people with a history of other factitious diseases. It can also happen secondary to a pharmacy error. Patients with this disorder will have increased measured insulin without the physiologic corresponding increase in C-peptide. Choice A, alimentary hypoglycemia, is a cause of hypoglycemia in patients with a history of gastrectomy. Choice C, beta cell insulinoma would present with elevated levels of both insulin and C-peptide.

In taking the family history of your 25-year-old male patient, you discover that he has numerous relatives with breast and ovarian cancers. In the past, his mother received genetic counseling and testing for the BRCA1 and BRCA2 gene mutations and was found to be positive for a mutant allele. What is his chance that he carries the genetic mutation? A His chance is 0%, because this is not transmitted to men. B 25% C 50% D 75% E 100%

Explanation The Correct Answer is: C BRCA1 and BRCA2 gene mutations are expressed in the cells of breast and other tissue, where they help repair damaged DNA, or destroy cells if DNA cannot be repaired. If the BRCA1 or BRCA2 itself is damaged, damaged DNA is not repaired properly and this increases risks for certain cancers. These genes are inherited in an autosomal dominant manner so if his mother is positive (and his father is not), his risk is 50%, and he is also at increased risk for not only breast cancer, but also prostate, pancreatic, and other cancers.

A 63 year-old woman complains of fatigue, loss of appetite, a sore-red tongue, paresthesias of her feet and hands, and unsteadiness of her gait. Which of the following tests is be used to confirm the patient's suspected diagnosis? A Anti-intrinsic factor antibodies B Antiparietal cell antibodies C Coomb's test D Schilling Test E Serum folate levels

The Correct Answer is: A A diagnosis of pernicious anemia (PA) can be supported through the presence of anti-intrinsic factor antibodies. Antiparietal (B) cell antibodies aren't associated with the development of PA. The Coomb's test (C) is used in the evaluation of hemolytic anemias. Serum folate (D) levels are beneficial in evaluating macrocytic anemias, but will not establish a diagnosis of PA. The Schilling test (D) was once commonly used to diagnosis PA, but is no longer available due to lack of available radiolabeled human intrinsic factor.

A 58 year-old man with a hemoglobin of 11.1 mg/dL and hematocrit of 33% is noted to have a serum folic acid level of 0.2 ng/dL. Which of the following risk factors for folate deficiency should this patient be assessed for? A Alcoholism B Diabetes C Down's syndrome D Smoking E Vegetarian diet

The Correct Answer is: A Alcoholism and the nutritional status that results is a common cause of folate deficiency. Diabetes (B) and smoking (D) are not directly associated with folate deficiency. Down's syndrome (C) risk can be reduced through the administration of prenatal folic acid. A vegetarian (E) diet provides excellent sources of folate.

A 20-year-old male presents to the emergency department complaining of pain to the right shoulder region while playing basketball. He states that his arm was pulled back and rotated while he was moving forward, and then felt a popping sensation in the shoulder. Since then he has not been able to move the shoulder at all due to pain and immobility. It is suspected that he has a dislocation. Given this scenario, what would be the most likely type? What type of shoulder dislocation did he most likely experience? A Anterior B Inferior C Multidirectional D Posterior E Superior

The Correct Answer is: A All of the various types of dislocations mentioned above are possible, but anterior dislocations are by far the most common (>95%) and they are the most common of all joint dislocations. Most occur as a result of a fall or other traumatic event and they may become recurrent. The shoulder is most susceptible to an anterior dislocation when it is abducted and externally rotated. The shoulder joint is considered a very mobile joint, but this also renders it very susceptible to injury.

A 36-year-old female presents to your family practice office concerned about a breast lump she discovered in the shower last night. You obtain a detailed history and these are some of your findings: • G2P2 • LMP: 2 weeks ago, normal • Sister diagnosed with breast cancer at age 42. • Mother is currently being treated for ovarian cancer. • Father is of Ashkenazi Jewish ancestry. Which of the following is NOT an appropriate recommendation for this patient? A Enhanced surveillance, including mammography alternating with MRI, every six months. B Genetic counseling and testing for BRCA1 and BRCA2 status. C Referral to a breast surgeon for mastectomy D Chemoprophylaxis with an aromatase inhibitor weekly.

The Correct Answer is: A Because of her ethnicity (Ashkenazi Jewish descent) and family history (two first degree relatives with breast and ovarian cancers), this patient is at a higher than normal risk for breast and ovarian cancers. With a physical finding of a palpable breast mass, this is even more ominous. She should be referred for an immediate mammogram but not necessarily MRI - and have continued enhanced surveillance annually and not every 6 months regardless of the result - as well as a referral to a breast surgeon for immediate consideration of a biopsy. A mastectomy is a possible consideration but not mandatory. She is certainly an appropriate candidate for consideration of referral to a genetic counselor and genetic testing to ascertain her BRCA1 and BRCA2 status. She is also an appropriate candidate for chemoprophylaxis.

Which of the following joints has the lowest occurrence rate of osteoarthritis? A Elbows B Hands C Hips D Knees E Spine

The Correct Answer is: A Because the elbow is not a weight bearing bone, the rate of osteoarthritis in the elbow is considerably less that what is found in locations like the hips, knees and spine. The hands have one of the highest rates of occurrence of osteoarthritis, likely due to their near constant use and propensity for minor (or major) injury. When elbow arthritis does develop it is often post-traumatic osteoarthritis related to a significant injury in the past that disrupted joint surface integrity or as a result of rheumatoid arthritis, a systemic illness. Osteoarthritis of the elbow will generally present with pain, stiffness, and decreased range of motion. Osteophytes that form on the medial elbow might be implicated should neurological symptoms develop that correlate with ulnar nerve distribution as this nerve does pass in close proximity to the elbow on the medial side

A 58-year-old male comes to your primary care office complaining of sadness, insomnia, loss of appetite, weight loss, and feelings of guilt or hopelessness for the past month. On further questioning you find that his wife of 30 years died of breast cancer shortly before the onset of symptoms. Which of the following is the most likely diagnosis? A Bereavement B Dysthymia C Depressive episode D Depressed mood E Major depression

The Correct Answer is: A Bereavement (A) is the normal grief response experienced after a significant loss and includes all the symptoms of depression and, by definition, lasts no longer than two months, although many will suffer from some symptoms for longer. Dysthymia (B) is a longstanding depressed mood for at least two years but not meeting the full criteria for a depressive episode (C). Major depression (E) requires at least one depressive episode, which requires at least five of the symptoms, one of which is depressed mood (D).

A 28-year-old male is concerned because he has a friend who was diagnosed with chronic hepatitis C and wants to know if he is at risk for this disease as well. Which of the following answers elicited in your history would make you the most suspicious? A previous intravenous drug use B blood transfusion last year C monogamous relationship D recent negative HIV test E previous history of hepatitis A

The Correct Answer is: A Intravenous drug abuse accounts for over 50% of hepatitis C cases. Recent blood transfusions are unlikely to cause hepatitis C. Hepatitis C is more likely to be contracted in patients with multiple sexual partners, although the incidence of transmission via sexual intercourse is considered low. Hepatitis A is transmitted via the fecal-oral route and does not increase the chances of hepatitis C.

A 64-year-old female presents for an annual examination. She is 5'0" and weighs 92 pounds; compared to her examination 3 years ago she has lost an inch in height. After performing a dual-energy x-ray absorptiometry (DXA) scan the diagnosis of osteoporosis is confirmed. Which of the following supplements should be recommended? A Calcium carbonate 1,000 mg and vitamin D 800 international units daily B Calcium carbonate 500 mg and vitamin D 2,500 international units daily C Calcium carbonate 2,000 mg and vitamin D 500 international units daily D Calcium carbonate 800 mg and vitamin D 100 international units daily

The Correct Answer is: A Calcium supplementation provides beneficial effects on bone mass throughout postmenopausal life and may reduce fracture rates up to 50%. Postmenopausal women receiving supplemental calcium over a 3-year period in a placebo-controlled, randomized clinical trial had stable total body calcium and BMD in the lumbar spine, femoral neck, and trochanter compared with the placebo group. Recommendations for calcium carbonate are 1,000-1,500 mg once daily orally. Vitamin D increases calcium absorption in the gastrointestinal tract, making calcium more available for reabsorption and circulation. Recommendations for Vitamin D are 800-2,000 international units once daily orally. Calcium Carbonate less than 1,000 mg per day is not sufficient (B and D). Vitamin D levels less than 800 IU per day is not sufficient (C and D).

A 43 year-old woman is brought to the emergency department in critical care secondary to a traumatic brain injury. Which of the following best describes her Cheyne-Stokes respiratory pattern? A Alternating periods of shallow and deep breathing B Difficult or labored breathing C Difficult of labored breathing while supine D Periods of absent breathing E Sudden awakening due to shortness of breath

The Correct Answer is: A Cheyne-Stokes respirations are characterized by shallow breaths that increase in rate and depth followed by periods of apnea (A). Dyspnea is difficult or labored breathing (B) and if it occurs while supine (C) is termed orthopnea. Apnea is noted periods of absent breathing (D). Sudden awakening due to shortness of breath (E) is paroxysmal nocturnal dyspnea.

A 32-year-old female professional golfer presents with a 3-week history of pain along her thumb and down her wrist. She denies any trauma and states that it is aggravated with any movement of her wrist and thumb. Her physical examination is unremarkable. You perform the following test: This test requires her to cup her thumb in a closed fist and ulnar deviate, which reproduces her pain. Considering the suspected diagnosis based on the positive examination above, what are the affected anatomical structures? Extensor pollicis brevis and abductor pollicis longus B Flexor pollicis longus and flexor digitorum profundus C Extensor carpi radialis longus and brevis D Ulnar collateral ligament

The Correct Answer is: A De Quervain tenosynovitis is a common condition that occurs in patients who have experienced excessive use of the thumb or wrist. This is a tenosynovitis of the extensor pollicis brevis and abductor pollicis tendons (A), where the tendons lie in the groove of the radial styloid. The diagnosis of De Quervain tenosynovitis is supported by a history of pain in this location along with a painful range of motion of the thumb and further confirmation may be provided by a positive Finkelstein test. The flexor pollicis longus and flexor digitorum profundus (B) are affected in carpal tunnel syndrome. Extensor carpi radialis longus and brevis (C) are extrinsic extensor muscles of the hand. The ulnar collateral ligament (D) is injured in Gamekeeper's thumb.

Patients in which of the following age groups are least likely to experience a dislocation or sprain when a significant stress is placed on their joints? A 5-10 years old B 15-20 years old C 35-40 years old D 55-60 years old E 75-80 years old

The Correct Answer is: A Dislocations and ligamentous injuries are uncommon in prepubertal children as the ligaments and joints are quite strong as compared to the adjoining growth plates. Excessive force applied to a child's joint is more likely to cause a fracture through the growth plate than a dislocation or sprain.

A 23-year-old college basketball player twists her ankle while practicing. She explains the injury that is consistent with an inversion mechanism. Based on this history, what ligament would you expect to be the most likely injured in the ankle of this patient? A Anterior talofibular B Anterior tibiofibular C Calcaneofibular D Deltoid E Posterior talofibular ligament

The Correct Answer is: A Greater than 25,000 ankle sprains happen in the USA every day and the vast majority of those are inversion ankle sprains. The anterior talofibular ligament is the first, and often only, ligament damaged in inversion ankle sprains. As the force of the inversion increases, other lateral ankle ligaments can be involved. When the anterior tibiofibular ligament is involved, this is referred to as a high ankle sprain and such injuries generally have a prolonged recovery time. Calcaneofibular ligaments are generally the second most frequently injured of the lateral ankle ligaments and when injury occurs it is typically in combination with the anterior talofibular ligament. The deltoid ligament is a very strong ligament on the medial aspect of the ankle. Eversion stresses the deltoid ligament, but strong eversion forces are rare and when they do occur, an avulsion fracture of the medial malleolus is more likely than a significant ligament tear. The posterior talofibular ligament is one of the lateral ankle ligaments and can be injured in an inversion injury, but the rate of injury to this ligament lags far behind the anterior talofibular or calcaneofibular ligaments. (Sarwark, Ed., Essentials of Musculoskeletal Care, 4 th Edition, 2010)

A 12 year-old African-American male with a history of sickle cell disease presents to the clinic for routine follow up. A peripheral smear is performed. Which of the following best describes the cause of the abnormality pictured in the patient's peripheral smear? (note photo taken from figure 29-12 in Harrison's) A Absent or non-functioning spleen B Failure of nuclear maturation C Intravascular hemolysis D Lead intoxication

The Correct Answer is: A Howell-Jolly bodies are noted in the peripheral smear and result from the lack of removal of nuclear material due to an absent or non-functioning spleen. Failure of nuclear maturation (B) produces macrocytosis, intravascular hemolysis (C) leads to the creation of schizocytes, lead intoxication (D) and thalassemia cause basophilic stippling, and uremia (E) is associated with Burr cells.

A 67 year-old with chronic kidney disease presents to the clinic with a hemoglobin of 8.6 mg/dl and hematocrit of 24%. Which of the following agents, if administered, can further increase this patient's risk of thrombotic vascular events? A Cyanocobalamin B Erythropoietin C Ferrous sulfate D Folic acid E Niacin

The Correct Answer is: B Erythropoietin (B) increases red blood cell mass and toxicity may lead to increased blood viscosity and potential thrombotic events.

What age group is most at risk to develop osteoid osteoma? A Adolescents B Elderly C Middle-aged D Neonates E Toddlers

The Correct Answer is: A Osteoid osteoma is a benign bone forming tumor that usually develops during a patient's second decade of life. This type of tumor is much more common in boys than girls and typically affects the lower extremities (femur and tibia primarily) and spine more than other areas of the body. Patients typically present with gradually progressive bone pain that is worse at night and does not correlate with activity level. The tumor produces high levels of prostaglandins, so symptoms usually improve in 20-25 minutes if the patient takes a medication like ibuprofen, ASA or other NSAIDS that are prostaglandin inhibitors. A lack of improvement in symptoms with these medications should lead health care providers to consider a different diagnosis. The pain of this condition may cause those afflicted in a leg to limp and have swelling, muscle atrophy or contractures and exquisite point tenderness. The condition usually resolves on its own over time, but symptomatic patients may require surgical resection or radioablation of the tumor. (Tochia, Editor, UpToDate, 2010)

A 32-year-old man presents to the urgent care center with a concern of scrotal tenderness that began 3 days ago and has now worsened. Physical examination reveals a temperature of 100.7°F, positive tenderness in the posterolateral aspect of the right testis with swelling, spermatic cord tenderness with palpation, and no transillumination. What is this patient's most likely diagnosis? A epididymitis B orchitis C epididymo-orchitis D testicular torsion

The Correct Answer is: A Pain and swelling are prominent features of epididymitis; fever and abdominal pain may also be present. Epididymitis is caused by an ascending infection that without treatment will continue to the testicles, causing a significant swelling that will make it difficult for the clinician to distinguish between the epididymis and the testicles (epididymo-orchitis). Orchitis alone is most commonly viral (mumps) and observed in prepubertal boys. In men younger than 30 years, epididymitis can be confused with torsion

A 66-year-old female is admitted to the hospital with atrial fibrillation. Her past medical history has hypertension, diabetes mellitus type II, hypercholesterolemia, and rheumatoid arthritis. During her evaluation for work up it is noted that the patient has had recurring symptomatic episodes of atrial fibrillation for the last year, some resulting in the patient not being able to ambulate due to hypotensive events. Her current blood pressure is 146/83, and her heart rate is 87. Given this clinical scenario, what is the most appropriate procedure for this patient? A Ablation therapy B Cardiac catheterization C Synchronized cardioversion D Permanent pacemaker insertion E Pacemaker and defibrillator insertion

The Correct Answer is: A The clinical scenario for patients who have recurrent symptomatic episodes related to atrial fibrillation is ablation therapy (A). Cardiac catheterization (B) is sometimes performed based on suspicion of coronary artery disease, but does not address the electrophysiological reasons for the patient's episodes. Cardioversion (C) would not address a patient who is having recurrent episodes, and there is no clinical indication for pacing or defibrillation needs (D and E).

A 23-year-old male presents to the clinic complaining of left anterior neck pain that developed over the past week following recovery from an acute upper respiratory infection. On physical exam a tender mass is felt anterior to the left sternocleidomastoid muscle from the mandible inferiorly to the level of the cricoid cartilage. Which of the following is the most likely diagnosis? A Branchial cleft cyst B Dermoid cyst C Peritonsillar abcess D Salivary gland tumor E Thryoglossal duct cyst

The Correct Answer is: A The development of a neck mass in a young adult following URI is consistent with branchial cleft cyst (A) and thyroglossal duct cyst (E). The location of this mass away from the midline and anterior to the SCM is most consistent with branchial cleft cyst (A).

Upon testing a patient for function of the hip extensors, which muscle is considered the primary muscle responsible for most extension? A Gluteus maximus B Pectineus C Semimembranosus D Semitendinosus E Vastus lateralis

The Correct Answer is: A The gluteus maximus is a large muscle that is partially responsible for giving shape to the buttocks. It is the dominant muscle responsible for hip extension. It is easily palpable with a patient in the prone position with buttocks squeezed together or with the hip extended and the knee flexed. The pectineus muscle is considered a secondary hip adductor. The Semimembranosus and Semitendinosus are two of the three hamstring muscles. They are primary movers in knee flexion, but only secondary contributors to hip extension. Vastus lateralis is one of the four quadriceps muscles and plays a role in knee extension, but not hip extension.

A 58 year-old male presents with a history of dyspnea on exertion and chronic cough worse with arising in the mornings. He has a 60-pack year history of cigarette use. On examination there is increased AP diameter and decreased breath sounds with a prolonged expiratory phase. Pulse oximetry reveals an oxygen saturation of 93% on room air. Current medications include varenicline and ipratropium. Which of the following is the most appropriate intervention at this time? A Influenza and pneumococcal vaccine B Montelukast C Oxygen therapy D Prednisone E Prophylactic antibiotic therapy

The Correct Answer is: A The patient has chronic obstructive pulmonary disease being treated with bronchodilators (ipratropium) and he is undergoing tobacco cessation therapy (varenicline). Additional health maintenance requirements include administration of influenza and pneumococcal vaccine (A). Antibiotics (E) are beneficial during acute COPD exacerbations. Oxygen therapy (C) is not required as evidenced by his pulse oximetry of 93%. Prednisone (D) is used in later stage disease that fails to respond to additional inhaled medications (long-acting beta agonists, corticosteroids). Montelukast (B) is used in the management of asthma.

A 73-year-old, male with a 30 pack-year smoking history presents to the clinic with complaints of headache and dizziness. Labs reveal a hemoglobin of 21.3 g/dl, hematocrit of 63%, and platelet count of 498,000. He tests positive for the JAK2 mutation. Which of the following should be recommended to this patient to prevent secondary complications from his diagnosis? A Aspirin 81 mg daily B Ferrous Sulfate 325 mg twice daily C Eat a diet rich in vitamin B12 D Eat a diet rich in folate E Maintain stable consumption of foods that contain vitamin K

The Correct Answer is: A The patient most likely has polycythemia vera and is at risk for thrombotic events that can be reduced through the use of daily aspirin. In addition, he should be counseled on smoking cessation if he is still currently smoking.

A 23-year-old man presents to the outpatient clinic for follow-up from a recent urgent care visit. He complains of sore throat, fever, fatigue, myalgias, and a rash that started 5 days ago, and have worsened since he was seen in the urgent care 3 days ago. The patient appears non-toxic with a temperature of 39.4 degrees Celsius. Physical exam reveals pharyngeal and tonsillar erythema without exudates, generalized lymphadenopathy, a morbilliform rash on his trunk, and no hepatosplenomegaly. A rapid strep screen and Monospot performed at the local urgent care were reportedly negative. Which of the following prevention strategies should be recommended to this patient? A Abstain from sexual activity B Avoid aspirin C Avoid contact sports and rest D Bedrest and increased fluids E Take the full course of antibiotics

The Correct Answer is: A The patient presentation is consistent with acute retroviral syndrome. The patient is highly contagious and should be counseled on strategies to prevent transmission of HIV to others

A 43-year-old female presents to the outpatient clinic complaining of itching and irritation of her right eye. She denies decreased vision or photophobia. On physical exam the patient's eye has the following appearance: Which of the following is the most appropriate management for this patient's condition? A Daily lid cleansing and application of bacitracin ophthalmic ointment 500 units/g B Doxycycline 100 mg by mouth once daily C Incision and drainage D Referral to an ophthalmologist E Warm compresses every 3 hours until resolution

The Correct Answer is: A The patient's symptoms of itching and irritation of the lid margin with an inflamed eyelid and eyelash scaling is consistent with mild anterior blepharitis, which is initially treated with cleansing and the potential addition of a topical antistaphlococcal antibiotic (A). Answers (C) and (E) are appropriate treatments for a hordeolum. Doxycycline once daily (B) can be used as a long-term treatment for posterior blepharitis. The condition doesn't warrant referral (D).

A 28-year-old male presents to the outpatient clinic with complaints of a painful, red right eye, with profuse drainage that started 2 days ago. He has increased tearing and photosensitivity. Physical exam reveals the following appearance of the upper tarsal conjunctiva. Which of the following is the most likely causative agent for this patient's conjunctivitis? A Chlamydia trachomatis B Herpes simplex virus C Neisseria gonorrhea D Pseudomonas E Streptococcus pneumoniae

The Correct Answer is: A The presence of acute follicular conjunctivitis is most consistent with inclusion conjunctivitis resulting from chlamydial infection (A). Follicles are most often seen in viral conjunctivitis and some forms of parasitic conjunctivitis, but not typically seen in bacterial conjunctivitis (C, D, and E).

A 23-year-old female presents to the outpatient clinic with complaints of a painful, red right eye, with profuse drainage that started 2 days ago. She has increased tearing and photosensitivity. Physical exam reveals the following appearance of the upper tarsal conjunctiva. Which of the following is the most appropriate treatment for this patient? A Azithromycin 1 gram by mouth B Ceftriaxone 1 gram IM C Olopatadine 0.1% ophthalmic solution twice daily D Sulfacetamide 10% ophthalmic solution three times a day for 5 days E Valacyclovir 500 mg by mouth twice daily for 10 days

The Correct Answer is: A The presence of acute follicular conjunctivitis is most consistent with inclusion conjunctivitis resulting from chlamydial infection. Follicles are most often seen in viral conjunctivitis and some forms of parasitic conjunctivitis, but not typically seen in bacterial conjunctivitis. The first-line treatment for inclusion conjunctivitis is azithromycin 1 gram orally (A).

A 56-year-old male patient is diagnosed with prostatitis. Which of the following is the least appropriate antibiotic to prescribe in an outpatient- family practice setting? A ceftriaxone B doxycycline C levofloxacin D trimethoprim-sulfamethoxazole (TMP-SMX) E All are appropriate antibiotics for this patient in this setting.

The Correct Answer is: A The response to antibiotics in acute bacterial prostatis is usually prompt, perhaps because drugs penetrate readily into the acutely inflamed prostate. Antibiotic selection should be guided by results of urine cultures and susceptibility results. Appropriate empiric antibiotics include a fluroquinolone (i.e.levofloxacin 500 mg once daily) or TMP/SMX (one double-strength tablet every 12 hours). Patients who are too ill for oral therapy or are septic on presentation should be hospitalized for initial parenteral treatment (intravenous quinolones with or without an aminoglycoside). Ceftriaxone would not be recommended as first-line.

A 23-year-old patient who has recently been on a ski trip presents with pain to the right hand after sustaining a fall. It is difficult to move, and there is pain on movement of the first digit. Based on this history what ligament would the patient most likely have injured? A 1st MCP joint ulnar collateral ligament B 2nd MCP joint ulnar collateral ligament C 3rd MCP joint ulnar collateral ligament D 4th MCP joint ulnar collateral ligament E 5th MCP joint ulnar collateral ligament

The Correct Answer is: A The ulnar collateral ligament at the base of the thumb, or 1st MCP joint, is often injured in forced abduction, such as a fall while skiing or during other sporting activities. An injury to this ligament has traditionally been called Gamekeeper's Thumb, but the origin of this term referred to a more chronic injury sustained by English gamekeepers as a result of the way they killed rabbits using their hands. Any of the MCP joint ulnar collateral ligaments could be injured in a fall if the mechanism of injury creates significant forces on the ligaments, but the 1st MCP joint is far more commonly injured than the others mentioned above.

An 8-year-old male presents to his primary care provider with the onset of a new rash, consisting of small, oval, discrete scaling plaques on his trunk, and a large red plaque with overlying thin, silvery scales in the gluteal cleft. Which of the following is a potentially important historical finding in this patient? A a recent history of group A strep infection B a family history of atopy C exposure to nickel in clothing D a personal history of allergies

The Correct Answer is: A This is a classic guttate psoriasis. An acute strep infection is a known precipitating factor of guttate psoriasis. All patients need to be checked and treated for a strep infection. Atopy has no correlation with guttate psoriasis. It is not caused by contact with an allergen or irritant. It is also not caused by an allergic reaction.

A 6-week-old male with sickle cell disease presents to the pediatric office for his well-child visit. When should this child begin taking daily prophylactic penicillin? A Now B 2 months C 6 months D 12 months E 5 years

The Correct Answer is: B Patients with sickle cell disease develop functional asplenia as early as 3 months of age and should begin treatment with prophylactic penicillin at 2 months of age to prevent infection by encapsulated organisms (i.e., pneumococcus).

In your family practice, you perform an ankle brachial index (ABI) on your 66-year-old diabetic who smokes with the results being 0.71 on the left and 0.68 on the right. Which of the following is the most appropriate next step? A Begin the patient on aspirin 81 mg and clopidogrel 75 mg daily. B Begin the patient on prasugrel 60 mg loading dose followed by 10 mg daily and refer to cardiology. C Begin the patient on a low-molecular weight heparinoid (LMWH) and refer to a vascular surgeon for further evaluation. D Begin the patient on warfarin 5 mg daily and titrate to an INR of 2.0 to 3.0.

The Correct Answer is: A This patient has peripheral arterial disease (PAD). This can be treated with antiplatelet agents, including aspirin and/or clopidogrel. Warfarin is an anticoagulant and is not FDA-approved for use in PAD. Your other consideration is referral to a cardiologist and/or vascular surgeon for further evaluation, depending upon the degree of symptoms. Patients with PAD have approximately one in six chance of have significant atherosclerosis in at least one other vascular bed, including carotids and coronary, and with a diabetic history, aggressive treatment of all co-morbidities could be life-saving. Effient, LMWH, and warfarin are not FDA-approved for treatment of PAD.

A 51-year-old male patient presents to your family practice office complaining of genital discomfort with dysuria. His digital rectal exam reveals an enlarged, tender prostate. His prostate-specific antigen (PSA) returns elevated with a value of 11.1 mg/mL, which you fractionate, and this reveals approximately 75% free PSA. His urinalysis reveals moderate white cells and trace blood. What would be your next step in treating this patient? A Begin him on 6 weeks of doxycycline to treat his prostatitis and when resolved, repeat his PSA level. B Immediately refer him to a urologist for prostate biopsy to rule out prostate cancer. C Immediately refer him to a urologist for cystoscopy to rule out bladder cancer and perform a computed tomography (CT) scan of the abdomen and pelvis in the interim. D Order a stat testicular sonogram to rule out torsion. E Order a CT scan of the abdomen and pelvis.

The Correct Answer is: A This patient has signs and symptoms consistent with prostatitis. Additionally, while his PSA is elevated, this is common in prostatits as well as prostate cancer, and his free PSA is of a percentage that prostate cancer is unlikely. However, it would be prudent to recheck his PSA after treatment and resolution of his symptoms to confirm that an underlying cancer is not smoldering.

What diagnosis should be given to a patient who has nonbizarre delusions for at least a month and no other symptoms? A schizoaffective disorder B delusional disorder C brief psychotic disorder D schizophreniform disorder

The Correct Answer is: B A delusional disorder presents with nonbizarre delusions for at least a month. The disorder does not present with any other symptoms related to schizophrenia or a mood disorder. A brief psychotic disorder has symptoms that last for 1 day to 1 month. Schizophreniform has symptoms that last at least a month, but no longer than 6 months. In schizoaffective disorders, depression or mania develop along with schizophrenic symptoms.

A 64-year-old female presents to your family practice office with a complaint of shortness of breath, dizziness, and fatigue over the past seven days. She states that she has experienced a similar episode twice in the past seven months, but it has resolved spontaneously in the past. However, this time she states it is different and is concerned about her condition. Her vital signs are stable and ECG is below. What is your diagnosis and which of the following is the appropriate initial treatment? A paroxysmal supraventricular tachycardia; start her on metoprolol ER 50 mg once daily and refer her to cardiology B persistent atrial fibrillation with a controlled rate; start her on warfarin 5.0 mg and check her INR in 5 days C sinus arrhythmia; reassure her that this is perfectly normal, associated with respirations, and that nothing further needs to be done D tell her that her ECG shows some irregularity; order a 24-hour holter monitor E she is having an anterior-inferior wall myocardial infarction; give her a full-dose aspirin and call 911

The Correct Answer is: B Atrial fibrillation is the most common chronic arrhythmia, with an incidence and prevalence that rises with age, so that it affects approximately 10% of individuals over 80 years old. It occurs in rheumatic and other forms of valvular heart disease, dilated cardiomyopathy, atrial septal defect (ASD), hypertension, and coronary heart disease. It also occurs in patients with no apparent cardiac disease; it may be the initial presenting sign in thyrotoxicosis, and this condition should be excluded with the initial episode. The heart rate may range from quite slow to extremely rapid, but is uniformly irregular unless there is a underlying complete heart block and a permanent ventricular pacemaker is in place. The surface ECG typically demonstrates erratic, disorganized atrial activity between discrete QRS complexes occurring in an irregular pattern. The atrial activity may be very fine and difficult to detect on the ECG, or quite course and often mistaken for atrial flutter. Atrial fibrillation often appears paroxysmally before becoming the established rhythm.

A 35-year-old male presents with pain and decreased range of motion after sustaining a fall in which the patient tried to grab onto a bar which pulled his entire arm in the process. Given this clinical scenario, at what cervical motor neuron level would the biceps reflex be testing? A C4 B C5 C C6 D C7 E C8

The Correct Answer is: B C5 is the primary motor neuron being tested in a biceps reflex. C6 contributes to the brachioradialis reflex primarily, but does have a small role in the biceps reflex. The C7 motor neuron is primarily involved in the triceps reflex. C4 and C8 do not contribute to any primary reflexes. (Hoppenfeld, Physical Examination of the Spine and Extremities, 1976)

Which of the following is a risk factor for cholesteatoma formation? A Acute otitis media B Chronic tympanic membrane retraction C Diabetes mellitus D Hyperlipidemia E Smoking

The Correct Answer is: B Cholesteatomas result from negative pressure in the middle ear chronically retracting the pars flaccida (B). Acute otitis media (A) may lead to short-term TM retraction or perforation, but without perforation or the placement of PE tubes it is unlikely to lead to cholesteatoma formation. Diabetes (C), hyperlipidemia (D), and smoking (E) are not associated with this localized pathology.

A 52-year-old female with a past medical history of dyslipidemia, GERD, hypothyroidism, and osteoarthritis presents with a gradual progression of fatigue and pallor over the last few months. Initial CBC results show a hemoglobin of 10.7 mg/dL, hematocrit of 33%, an MCV of 117 fL, and a reticulocyte count of 0.0%. Which of the following medications should be considered as a potential cause of her anemia? A Celecoxib B Cholestyramine C Levothyroxine D Antacids E Ranitidine

The Correct Answer is: B Cholestyramine is associated with folate malabsorption and may rarely be a medication cause for macrocytic anemia. Celecoxib (A), levothyroxine (C), antacids( D), and ranitidine (E) are not strongly associated with vitamin B 12 or folate deficiency leading to macrocytic anemia

A 43 year-old woman undergoes pulmonary function tests (PFTS) to evaluate progressive dyspnea and cough. The diffusion capacity is noted to be significant reduced. What is the most likely type of lung disease this patient is experiencing? A Congenital B Interstitial C Obstructive D Restrictive E Fixed extra-thoracic obstruction

The Correct Answer is: B Diffusion capacity is reduced in conditions that effect alveolar gas exchange such as, interstitial lung disease (B), atelectasis, pneumonia pulmonary vasculature disease, and late stage emphysema (due to destruction of alveoli, not obstructive lung disease).

According to the ATP III revised guidelines, which of the following should be your primary focus of treatment in patients with dyslipidemia? A lowering apolipoprotein B B lowering LDL cholesterol C Lowering non-HDL cholesterol D lowering triglycerides E raising HDL cholesterol

The Correct Answer is: B Given their proven efficacy, ease of administration, and enhanced patient compliance over other classes of medications, statin agents are the drugs of first choice for most patients. In particular, patients with diabetes or those in the highest risk category derive special benefits from their use due to their innate anti-inflammatory effects. Myopathy and elevated liver enzymes are the main potential side effects from statin agents. An increase of serum aminotransferase levels to more than three times normal occurs in 1% of patients taking high doses of statins. Monitoring of liver function tests at six weeks, 12 weeks, six months, and annually thereafter can help identify patients with hepatic side effects and facilitate prompt discontinuation of the agents. Rhabdomyolysis occurs in less than 0.1% of cases. It can be prevented by the prompt discontinuation of the agent when muscle pain and elevated muscle enzymes occur. Unexplained pain in large muscle groups should prompt investigation for myopathy; however, routine monitoring of muscle enzymes is not supported by any evidence. Side effects from statins may not be class specific. Therefore, a side effect with one agent should not prevent a trial with another statin agent. Prior concerns about statins causing cataracts or cancer have been alleviated by the release of two large meta-analyses in 2001.

A 76 year-old woman with steroid dependent chronic obstructive pulmonary disease is hospitalized with fever, chills, and a productive cough. The sputum gram stain shows many WBCs and small, pleomorphic gram-negative rods. Which of the following is the most likely causative agent? A Chlamydia pneumoniae B Haemophilus influenzae C Mycoplasma pneumoniae D Staph aureus E Strep pneumoniae

The Correct Answer is: B Haemophilus influenzae (B) is a gram-negative pleomorphic coccobacillus. Strep pneumonia (E) and Staph aureus (D) are gram positive organisms. Mycoplasma pneumonia (C) and Chlamydia pneumoniae (A) aren't visible on gram stain.

A 34-year-old male presents to the primary care office with a complaint of heartburn that has been present for three months. He has symptoms two to three times a week, which occurs about 30 minutes after eating. He has tried over-the-counter antacids and they were helping to relieve his symptoms for a few months, but they are not working well now. He denies dysphagia, odynophagia, or weight loss. You decide to treat him with a proton pump inhibitor at this visit, and he achieves good symptomatic relief with this therapy. What length of therapy is appropriate in this patient? A Two to four weeks B Eight to twelve weeks C Four to six months D One year E Continue indefinitely

The Correct Answer is: B If a patient achieves good symptomatic relief with a course of an empiric, once-daily proton pump inhibitor, therapy may be discontinued after eight to twelve weeks.

A 16-year-old girl presents to the clinic complaining of strong desires to sleep at inappropriate times. She is very concerned because she "felt paralyzed" while falling asleep on the couch last night. Which of the following is the best diagnostic test to confirm this patient's diagnosis? A CT of the head B multiple sleep latency test C Tensilon test D thyroid stimulating hormone E polysomnography

The Correct Answer is: B Narcolepsy is characterized by hypersomnolence, loss of muscle tone prior to sleep, hallucinations upon initiating or arising from sleep, and episodes of sleep paralysis. The diagnostic test that is used in conjunction with clinical history to establish the diagnosis is the multiple sleep latency test. The Tensilon test is utilized to assess for the presence of myasthenia gravis. Polysomnography can be useful in excluding other sleep disorders, but it does not assess sleep latency time necessary to support the diagnosis of narcolepsy.

Which of the following diseases is associated with the development of nasal polyps? A Amyloidosis B Allergic rhinitis C Chronic sinusitis D Chronic obstructive pulmonary disease E Wegener's granulomatosis

The Correct Answer is: B Nasal polyps are most commonly idiopathic although they may develop secondary to allergic rhinitis (B), or cystic fibrosis. Chronic sinusitis (C) may result from obstruction of the sinus drainage secondary to a polyp. Amyloidosis (A) and Wegener's granulomatosis (E) lead to the development of lesions with histology and appearance different than a benign nasal polyp.

You are evaluating a 67-year-old female who has history and physical exam findings consistent with osteoarthritis. Based on your knowledge of the disease, which joints are most commonly affected? A Carpometacarpal (CMC) B Distal interphalangeal (DIP) C Metacarpophalangeal (MCP) D Proximal interphalangeal (PIP) E Radiocarpal

The Correct Answer is: B Osteoarthritis can affect all of the joints mentioned, but generally has the highest prevalence in the DIP joints, especially the second DIP joint. Osteophyte formation at the DIP joints produce enlargements referred to has Heberden's nodes. The first carpometacarpal joint is likely the second most commonly affected joint. This painful and potentially debilitating condition at the base of the thumb can make grasping activities difficult and be exacerbated by prolonged or strenuous use of the thumb. While metacarpophalangeal and proximal interphalangeal joints can definitely be damaged in osteoarthritis, they are the more classic locations for rheumatoid arthritis changes to be manifested. Radiocarpal joints at the wrist are susceptible to osteoarthritis, but not at the same high frequency as the DIP joints. (Sarwark, Ed., Essentials of Musculoskeletal Care, 4 th Edition, 2010)

How many types of primary osteoporosis are there? A 1 B 2 C 3 D 4 E 5

The Correct Answer is: B Osteoporosis is a condition characterized by low bone mass, which increases the fragility of bones and leads to an increased risk of fracture. Osteoporosis is defined as being either primary or secondary and primary osteoporosis is further broken down into Type 1 and Type 2. Type 1 is related to decreased hormone levels - estrogen in women and testosterone in men, and is sometimes referred to as "postmenopausal osteoporosis." It is six times more common in women and results in loss of trabecular bone. Type 1 primary osteoporosis often presents with vertebral compression fractures or fractures of the distal radius after a fall. Type 2 primary osteoporosis is sometimes referred to as "senile osteoporosis" and generally occurs in patients over 70 years of age. It is twice as common in women as men and occurs due to a diminished capacity to make new bone. The most common types of fractures found in this type of osteoarthritis are hip and pelvic fractures. Secondary osteoporosis occurs at a somewhat higher rate in men versus women and is caused by some other medical condition that produces bone loss. Common causes include long-term steroid use, various endocrine abnormalities, and neoplastic diseases such as multiple myeloma. (Sarwark, Ed., Essentials of Musculoskeletal Care, 4 th Edition, 2010)

A 16-year-old female patient presents to her family practice office for her first dose of the quadravalent human papillomavirus (HPV) vaccine. As her physician assistant, when would you schedule her to come in for her second dose of the HPV vaccine? A two weeks B two months C three months D four months E six months

The Correct Answer is: B People are frequently confused by differences between the quadravalent and bivalent versions of this vaccine. But dosing schedules are not one of the confusing issues. The Centers for Disease Control and Prevention (CDC), Advisory Committee on Immunization Practices (ACIP), and the manufacturers of both the HPV4 and the HPV2 vaccines all agree: The dosing and administration schedules are the same for HPV4 and HPV2. Each dose is 0.5 mL, administered intramuscularly, preferably in a deltoid muscle. The vaccines are administered in a three-dose schedule. The second dose is administered one to two months after the first dose, and the third dose is administered six months after the first dose.

A 43 year-old woman recently diagnosed with sarcoidosis returns to the clinic for follow-up. Her chest x-ray demonstrates bilateral hilar lymphadenopathy. What stage of disease is this finding most consistent with? A Stage 0 B Stage 1 C Stage 2 D Stage 3 E Stage 4

The Correct Answer is: B Stage 1 (B) disease consists of bilateral hilar and/or mediastinal adenopathy without pulmonary parenchymal abnormalities. Stage 0 (A) is a normal chest x-ray. Stage 2 (C) includes adenopathy and lung parenchymal abnormalities. Stage 3 (D) includes diffuse lung parenchymal disease without nodal enlargement, and Stage 4 (E) is end-stage lung disease with pulmonary fibrosis and honeycombing.

A 23-year-old patient presents to the Emergency Department complaining about a bloody left eye that occurred after being struck with a tennis ball. The patient denies any decreased vision or photophobia. A photo of the left eye reveals the following (see below). Which of the following is the most appropriate management for this patient? A CT scan of orbit B Observation and reassurance C Olopatadine 0.1% ophthalmic solution twice daily D Refer to ophthalmology E Sulfacetamide 10% ophthalmic solution three times a day for 5 days

The Correct Answer is: B The classic appearance of a bright red, flat collection of blood is consistent with a subconjunctival hemorrhage best managed with observation and reassurance (B) by the primary care provider. The mechanism of injury is inconsistent with orbital fracture (A), and the patient lacks infectious (E) or allergic (C) conjunctivitis.

A 23-year-old male with a recent diagnosis of Non-Hodgkin's lymphoma presents complaining of swelling of the neck and face, cough, and dyspnea on exertion. On exam you note dilated neck veins. Which of the following is the most appropriate initial treatment for this patient? A Beta-blockers B Glucocorticoids C IV fluids D Trendelenburg position E Vena cava stenting

The Correct Answer is: B The classic presentation signs and symptoms of superior vena cava syndrome (SVCS) are present in this patient. Patients with lymphoma often respond with a decrease in tumor mass and improvement in SVCS with glucocorticoid therapy.

A disk herniation that is putting pressure on the L5 nerve root may present with weakness of what muscle(s)? A Anterior tibialis B Extensor hallucis longus C Gastrocnemius-soleus D Iliopsoas E Peroneus longus and brevis

The Correct Answer is: B The extensor hallucis longus muscle's motor function is associated the L5 motor neuron, which also supplies the gluteus medius and extensor digitorum longus and brevis muscles. The anterior tibialis muscle is supplied by the L4 motor neuron. Nerves emanating from T12, L1, L2 and L3 supply the iliopsoas. Gastrocnemius, soleus and peroneus longus and brevis are all supplied by nerves coming from the S1 area. The plantar flexing gastrocnemius and soleus muscles also are supplied by S2.

A 5-year-old male is being evaluated for an acute injury to the right ankle. On the x-ray of the ankle there is a distal tibia fracture that involves the separation of the growth plate, as well as a small non-displaced chip fracture of the metaphysis of the tibia. Based on these findings, what type of Salter-Harris fracture does this child have? A I B II C III D IV E V

The Correct Answer is: B The growth plate is the most fragile part of the bone prior to bone maturation and thus is usually the first structure disrupted when force is applied. Statistically, Type II fractures are most common - those that involve both the growth plate and a chip fracture of the metaphysis. (R

Upon testing a patient for function of the hip flexors, which muscle is considered the primary muscle responsible for most flexion? A Gracilis B Iliopsoas C Rectus femoris D Sartorius E Vastus intermedius

The Correct Answer is: B The iliopsoas muscle is the primary hip flexor muscle. It originates at T12 and L1-5 vertebrae and intervertebral disks as well as the iliac fossa of the pelvis and connects to the femur at the lesser trochanter. The gracilis muscle is considered a secondary hip adductor. Rectus femoris does help with hip flexion, but in a secondary role to the iliopsoas. Rectus femoris is also involved in knee extension. Sartorius is also involved in hip flexion, but in a secondary role. The vastus intermedius muscle is one of the four quadriceps muscles and is involved with knee extension and is not involved in hip flexion.

Which peripheral nerve is involved in the most common compression neuropathy in the upper extremity? A Axillary B Median C Radial D Sciatic nerve E Ulnar

The Correct Answer is: B The median nerve is commonly compressed as it passes through the carpal tunnel in the wrist

A 48-year-old man presents to the clinic for a routine employment physical. The patient is asymptomatic. A pre-employment CBC reveals hemoglobin of 13.2g/dl, hematocrit of 39.5%, and MCV of 60.6 fL. Subsequent iron studies, hemoglobin electrophoresis, and sickle cell screening are "normal." After the labs are reviewed, the patient states "they are always normal." What is the most likely diagnosis? A Anemia of chronic disease B Alpha thalassemia minor C Beta thalassemia major D Iron deficiency anemia E Sickle cell disease

The Correct Answer is: B The patient has a mild anemia with pronounced microcytosis consistent with alpha thalassemia minor. Alpha thalassemia minor hemoglobin electrophoresis reveals normal results. Patients with anemia of chronic disease (A) and iron deficiency anemia (D) have abnormal iron studies; while patients with beta thalassemia major (C) and sickle cell disease (E) have abnormal hemoglobin electrophoresis results.

A 29-year-old woman presents to the clinic with a complaint of severe diarrhea occurring over the last 3 to 4 days. Upon examination, the patient displays poor skin turgor and has a temperature of 100.2°F. In the supine position, the patient's blood pressure is 88/64 mm Hg and her heart rate is 112 beats/min. Upon standing, her heart rate further increases to 126 beats/min. Which of the following accounts for the further increase in the patient's heart rate upon standing? A decreased systemic vascular resistance B decreased venous return C increased preload D increased myocardial contractility E increased peripheral vasodilation

The Correct Answer is: B The patient is displaying signs of hypovolemia likely because of her chronic diarrhea. Upon standing, most of her low blood volume pools in the veins of her lower extremities because of the effects of gravity. As a result, even less blood returns to the heart, which leads to a decrease in both stroke volume and cardiac output as well as orthostatic hypotension. This elicits the baroreceptor reflex, which attempts to increase and maintain arterial blood pressure by raising the heart rate.

A 22 year-old male presents with a 2-week history of an upper respiratory infection that hasn't improved after taking amoxicillin for 6 days. He notes persistent sore throat, intermittent fever, and a worsening nonproductive cough. Physical examination reveals bilateral diffuse crackles. What is the most appropriate antibiotic to initiate after discontinuing the amoxicillin? A Amoxicilin and clavulanic Acid B Azithromycin C Cephalexin D Ciprofloxacin E Trimethoprim/Sulfamethoxazole

The Correct Answer is: B The patient most likely has atypical pneumonia that responds best to macrolide antibiotics (B). Antibiotics that inhibit cell wall synthesis (A, C) are generally ineffective against these atypical organisms that are either intracellular or lack a cell wall.

A 76 year-old man with long-standing asthma presents to the clinic complaining of increased use of his albuterol and 2-3 nighttime awakenings over the last month. Which of the following is the most appropriate therapy? A Beclomethasone via a pressurized meter dose inhaler B Fluticasone dry powder inhaler C Montelukast orally D Prednisone orally E Salmeterol dry powder inhaler

The Correct Answer is: B The patient requires inhaled corticosteroid therapy for moderate-persistent asthma. Fluticasone (B) dry powder inhaler is a preferred agent in this geriatric patient who is at increased risk of having poor inhaler technique. Beclomethasone (A) via a pMDI is prone to deposition in the oropharynx, decreased efficacy, and increased side effects. These concerns can be mitigated through the use of a spacer. Montelukast (C) and salmeterol (E) are not preferred agents for this stage of asthma.

A 43-year-old woman presents to the outpatient clinic complaining of right eye redness, photophobia, and pain. She notes some blurred vision and denies the presence of discharge. On physical exam her visual acuity is 20/20 left eye, and 20/60 right eye. Her right eye has circumcorneal injections and the pupil is 3 mm and responds poorly to light. Her left pupil is 5 mm and responds well. Fluorescein staining of the eye is unremarkable and intraocular pressures are normal. Which of the following treatment regimens should be prescribed? A Homatropine 5% solution four times daily B Homatropine 5% solution four times daily and prednisolone 1% solution every 1 or 2 hours while awake C Prednisolone 1% solution every 1 or 2 hours while awake D Prednisolone 1% solution every 1 or 2 hours while awake and sulfacetamide 10% solution three times a day E Prednisone 60 mg by mouth once daily

The Correct Answer is: B The patient's presentation of acute uveitis is best treated with topical corticosteroids and cycloplegics (B) once infectious causes (e.g., HSV) have been ruled out. The addition of a cycloplegic helps reduce pain. Antibiotic drops (D) aren't indicated for acute uveitis.

An 11-year-old boy is being seen in the clinic for well-child care. His father inquires whether his son is starting to show physical signs of puberty. Which of the following is the first sign of puberty in males? A change of voice B scrotal and testicular enlargement C gynecomastia D pubic hair development

The Correct Answer is: B s crotal and testicular enlargement is the first sign of puberty in boys. This typically occurs at the average age of 10 to 12 years.

What types of connective tissue are injured in a sprain? A Bones and muscles B Fascia and joint capsules C Ligaments and joint capsules D Muscles and tendons E Tendons and bones

The Correct Answer is: C A sprain involves injury to those tissues that give support to joints - ligaments and joint capsules. Injury to muscles, tendons, and fascia would all be classified as a strain. Injuries to bone would be classified as a fracture

A 23 year-old woman seeks guidance on foods, medications, and supplements that can help increase the amount of iron supplement. Which of the following is the best advice to provide this patient? A Co-administer with a calcium supplement B Co-administer with proton pump inhibitors C Co-administer with vitamin C D Taking with a basic beverage E Taking with food

The Correct Answer is: C An acidic achieved through presence of additional acids (C) (e.g., ascorbic acid) increases absorption of iron.

A 64-year-old African American female presents to the clinic for evaluation of her hypertension, which she has had for several years. In the past she had been taking hydrochlorothiazide and lisinopril, with little effect on her blood pressure management. At today's visit, she has no complaints and feels well. Her vitals show T m 96.6, P 85, R 18, BP 191/99. She has no jugular venous distention seen on the neck exam, her lungs are clear, and cardiac exam has a regular rate and rhythm without murmur or gallop. Her abdomen is soft, non-tender, and a bruit is appreciated at the mid-abdomen just a few centimeters below the epigastric region. There is no fullness or enlargement of the abdominal aorta on palpation. Based on the history and clinical findings, what is the most appropriate next test for this patient? A Computed tomography of the abdomen B Nuclear exercise stress test C Renal duplex ultrasound D Echocardiogram E Electrocardiogram

The Correct Answer is: C Based on the information of the presentation, the patient most likely has renal artery stenosis, which is causing uncontrolled hypertension. The most appropriate test for confirmation of this is a renal duplex ultrasound (C). This test will have a higher specificity and sensitivity than the other listed tests (A, B, D, and E), as well as determine flow.

Which condition is suggested by urethritis, arthritis, and conjunctivitis? A chlamydial infection B gonococcal infection C reactive arthritis D tertiary syphilis

The Correct Answer is: C Both chlamydia and gonorrhea infections can result in urethritis. Gonococci can disseminate to the joints and cause septic arthritis. Chlamydia is typically asymptomatic but can cause chronic conjunctivitis in adolescents and young adults. Reactive arthritis (also known as Reiter syndrome) is a result of an untreated chlamydia infection, and although typically characterized, in texts, by the triad of urethritis, arthritis, and conjunctivitis, all of the symptoms may not be present or not identified at the time of presentation. Tertiary syphilis is characterized by neurologic and cardiovascular disease, gumma, auditory and ophthalmic involvement, and cutaneous lesions. (Gravel et al., 2007, pp. 936-938) Gravel J , Comeau D , Gordon A. Rheumatology and musculoskeletal problems. In: Rakel RE, ed. Textbook of Family Medicine. 7th ed. Philadelphia, PA: Saunders; 2007:915-953.

A 45-year-old male comes into your family practice office for his second follow-up appointment since being diagnosed with type 2 diabetes. He does not complain of any symptoms. He is currently taking metformin 1000 mg BID and his HgA 1c at this visit 7.6%. You ask him about his diet and if he is regularly monitoring his blood glucose. He says that he does not really watch what he eats, but he does check his glucose levels daily. He tells you that on average his fasting plasma glucose (FPG) is usually around 88 mg/dl and his postprandial glucose (PPG) is around 180 mg/dl. What would be the BEST next appropriate step in management for this patient? A Nothing, his levels are within normal limits. B Consider switching to basal insulin therapy to control his FPG. C Add a dipeptidyl peptidase 4 (DPP-4) inhibitor to control his PPG. D Add a sulfonylurea to control his PPG. E Increase his metformin to 1500 mg twice daily.

The Correct Answer is: C DPP-4 inhibitors modulated glucagon-like peptide-1 (GLP-1). Their mechanism of action is thought to result from increased incretin levels, especially GLP-1. GLP-1 inhibits glucagon release which, in turn, results in increased insulin secretion, delays gastric emptying, and decreases serum glucose levels. The class is particularly appropriate to utilize in patients who have near-normal HbA 1C and elevated postprandial serum glucose because they work only when food enters the gut and have little to any chance of hypoglycemia. Sulfonylurea is more likely to result in hypoglycemia.

An 11-month-old African-American male presents to the pediatric office with lethargy, jaundice and splenomegaly. A CBC reveals hemoglobin of 8.0 mg/dl and a hematocrit of 25%. Peripheral smear appearance is available below. Which of the following treatments will reduce hemolysis and increase hemoglobin levels for this patient? A Allogeneic stem cell transplant B Folic acid supplementation C Hydroxyurea D Prophylactic penicillin E Pneumococcal vaccine

The Correct Answer is: C Hydroxyurea directly reduces hemolysis and increases levels of fetal hemoglobin and reduces complications and transfusion frequency. Sickle cell disease can be cured in 80% of individuals who receive a suitable transplant (A), but this procedure carries great risk and donors are difficult to identify. Folic acid supplementation (B) and pneumococcal vaccine (E) are indicated for preventive therapy of all patients with sickle cell disease, and prophylactic antibiotics (D) are used to prevent recurrent infections.

A diet high in nitrates is a significant risk factor for cancer of which of the following? A oropharynx B esophagus C stomach D pancreas E liver

The Correct Answer is: C In addition to chronic H pylori infections, dietary nitrates are a significant risk factor for gastric cancer

A 73-year-old female with type 2 diabetes, hypertension, and hyperlipidemia presents to the outpatient clinic complaining of left ear pain, and a yellowish-green, foul-smelling discharge that began about 3 weeks ago. On physical examination, the patient is afebrile and examination reveals a markedly edematous left ear canal draining purulent, green discharge. The tympanic membrane is unable to be visualized. Which of the following is the most likely diagnosis? A Auricular cellulitis B Cerebrospinal fluid leakage C Malignant otitis externa D Otitis externa E Serous otitis media with perforation

The Correct Answer is: C Malignant otitis externa (C) is a more serious form of otitis externa (D) that most commonly occurs in patients with diabetes and is most commonly caused by pseudomonas. The case scenario describes the location as the external auditory canal without significant involvement of the auricle (A). Serous otitis media with perforation (E) and CSF leak (B) would present with a clear drainage.

Which type(s) of Salter-Harris fractures can generally be treated with closed reduction and cast immobilization? A Type I B Types I and II C Types I, II, and III D Types I, II, III, and IV E Types I, II, III, IV, and V

The Correct Answer is: C Minimally displaced Salter-Harris types I, II, and III fractures generally can be treated with immobilization only. Types IV and V involve the cartilage of both the articular surface and the growth plate. To ensure proper alignment and a congruous joint surfaces open reduction and internal fixation is usually necessary. (Sarwark, Ed., Essentials of Musculoskeletal Care, 4 th Edition, 2010)

A patient presents to your family practice office with classic renal colic with his pain being a 6 on a 1 to 10 scale, with 10 being the worst pain imaginable. You refer your patient for a stat computed tomography (CT) scan of the kidneys. Your radiologist calls and advises that the CT reveals that your patient has a 4 mm stone at the ureterovesical junction (UVJ) on the left side without evidence of hydronephrosis. What should you advise your patient? A Go directly to the emergency department for admission. B Go directly to the emergency department for hydration and pain management. C Return to the office, administer ketorolac IM, give a prescription for an opioid, increase fluids, strain their urine, go to the emergency department if the pain worsens, and return in 48 hours for a reevaluation. D Go home, take four Motrin IB tablets every eight hours for pain, and go to the emergency department if the pain worsens. E Refer the patient for immediate lithotripsy.

The Correct Answer is: C Nonsteroidal anti-inflammatory drugs (NSAIDs) in general, and ketoralac specifically more so than other, causes constriction of the renal afferent arteriole, reducing pressure on a kidney stone and providing significant pain relief. Generally speaking a stone of less than 5 mm, especially at the UVJ, will pass relatively rapidly (within 48 hours) and the patient only needs to strain their urine to collect the stone and receive analgesia.

A 64-year-old postmenopausal female presents for an annual examination. She is 5'0" and weighs 92 pounds; compared to her examination 3 years ago she has lost an inch in height. After performing a dual-energy x-ray absorptiometry (DXA) scan the diagnosis of osteoporosis is confirmed and she is started on appropriate treatment. How often should she be recommended to follow-up with a DXA bone density scan? A Every six months B Every year C Every two years D Every ten years E No need to follow-up

The Correct Answer is: C Recommendations for patients diagnosed with osteoporosis without a fracture is every 2-3 years (C). This guideline is approved based on age, risk factors, or previous fractures. A post-menopausal woman at age 64 without a presenting fracture can be followed every 2-3 years with bone density imaging (DXA scan). Medications and other treatment options may need earlier or more frequent follow-up depending on the treatment plan. DXA is quite accurate and delivers negligible radiation. Performing a DXA at intervals less than two years has not shown of benefit (A and B). Every 10 years is too far of a time frame to follow osteoporosis (D). Follow-up is needed to see any changes in the bone density (E).

A 16-year-old girl is referred for a sports physical. Her blood pressure is 170/92 mm Hg. Urinalysis (UA) reveals 2+ protein. The girl's mother reports multiple episodes of urinary tract infections (UTIs) throughout childhood that were never investigated. The most likely diagnosis is A obstructive uropathy B orthostatic proteinuria C chronic reflux nephropathy D nephrotic syndrome E exercise-induced proteinuria

The Correct Answer is: C Retrograde flow of urine from the bladder damages the renal interstitium, causing inflammation and fibrosis. If untreated, irreversible damage to the kidneys will occur. Because this is a tubulointerstitial process, the urinalysis will be negative for protein in the early stages of damage. Most damage is done before age 5, but if undetected, glomerular damage will occur and protein will appear in the urine eventually. Hypertension develops as the GFR decreases.

A 23 year-old woman presents to the clinic complaining of abrupt onset of severe pallor, fatigue and dyspnea on exertion. Initial CBC reveals a hemoglobin of 9.6 mg/dL and an MCV of 87 fL. Which of the following best describes the cause of the abnormality pictured in the the patient's peripheral smear? (note photo taken from figure 29-13 in Harrison's) A Absent or non-functioning spleen B Failure of nuclear maturation C Intravascular hemolysis D Lead intoxication E Presence of uremia

The Correct Answer is: C Schizocytes are present on the peripheral smear and result from intravascular hemolysis. Howell-Jolly bodies result from the lack of removal of nuclear material due to an absent or non-functioning spleen (A). Failure of nuclear maturation (B) produces macrocytosis, lead intoxication (D) and thalassemia cause basophilic stippling, and uremia (E) is associated with Burr cells.

Radiculopathy due to nerve root compression occurs most commonly at which nerve root within the brachial plexus? A C5 B C6 C C7 D C8 E T1

The Correct Answer is: C The C7 nerve root is affected the most often (approximately 45-60%). This radiculopathy can result from foraminal encroachment of the spinal nerve, cervical disk herniation, tumor, and multiple sclerosis. C7 radiculopathy can present with weakness in the triceps, which cause elbow extension, and finger flexion and extension. C6 is another common site of radiculopathy. C6 radiculopathy can present with weakness in the biceps, brachioradialis, and wrist extensor muscles. Cervical radiculopathy at the C5, C8, and T1 are less common, but still possible. C5 radiculopathy can present with deltoid and biceps muscle weakness. C8 radiculopathy can present with finger flexor weakness and T1 radiculopathy with finger abduction weakness.

A 66-year-old female is admitted to the hospital with a new onset of atrial fibrillation. Her past medical history has hypertension, diabetes mellitus type II, hypercholesterolemia, and rheumatoid arthritis. Her vital signs show a blood pressure of 136/78, pulse of 89, respirations 18, and oxygen saturation of 96%. Her lungs are clear to auscultation, and an irregularly irregular rhythm is appreciated. There is no edema on peripheral examination. Given this clinical scenario, what is the most appropriate test in determining if an intracardiac thrombus is present? A Transthoracic echocardiography B Electrocardiogram C Transesophageal echocardiography D Cardiac catheterization E Magnetic resonance arteriography

The Correct Answer is: C The best and most appropriate test to evaluate for an intra-atrial thrombus is the transesophageal echocardiogram (C). This test has the better sensitivity and specificity when compared to the transthoracic echocardiogram (A). Electrocardiograms (B), cardiac catheterizations (D), and magnetic resonance imaging (E) do not give any specific information related to the evaluation of an intracardiac thrombus.

A 18 year-old man presents to the clinic with a lifelong history of transfusion dependent anemia and iron chelation therapy. Which of the following hemoglobin electrophoresis findings is most likely present? A Decreased HbA2 and increased HbF levels B Decreased HbA2 and HbF levels C Increased HbA2 and normal HbF levels D Increased HbA2 and decreased HbF levels E Normal HbA2 and HbF levels

The Correct Answer is: C The patient history is most consistent with beta thalassemia major that presents with increased HbA 2 and/or HbF levels. Alpha thalassemia is typically a less severe anemia and presents with a normal hemoglobin electrophoresis (E), since all adult hemoglobin are alpha-containing and affected equally. Beta thalassemia minor also typically presents with increased HbA 2 (C) and can typically be distinguished from beta thalassemia major by the clinical less sever clinical presentation.

A 78-year-old obese male smoker presents with a gradual progression of fatigue and pallor over the last few months. Initial CBC results show a hemoglobin of 10.4 mg/dL, hematocrit of 32%, an MCV of 74 fL, and a reticulocyte count of 0.1%. Iron studies reveal an elevated ferritin, decreased serum iron and TIBC. Which of the following is the most likely cause of this patient's anemia? A Alpha thalassemia minor B Beta thalassemia C Anemia of chronic disease D Hemolytic anemia E Iron deficiency anemia

The Correct Answer is: C The patient most likely has an underlying chronic disease resulting in anemia, as evidenced by his adequate iron stores and inability to properly use that iron for RBC production. Iron studies in patients with thalassemia (A and B) are typically normal. Iron deficiency anemia (E) is confirmed with low ferritin, serum iron, and increased TIBC.

A 28-year-old female presents with a gradual progression of fatigue and pallor over the last few months. Initial CBC results show a hemoglobin of 10.4 mg/dL, hematocrit of 32%, an MCV of 112 fL, and a reticulocyte count of 0.1%. Which of the following is the most likely pathophysiologic mechanism responsible for her anemia? A Acute blood loss B Defective bone marrow/stem cell function C Defective DNA production D Defective hemoglobin production E Increased destruction of red blood cells

The Correct Answer is: C The patient most likely has an underlying vitamin B 12 or folate deficiency resulting in macrocytosis (MCV of 112 fL).

A 15-year-old male presents complaining of a sore throat, headache, and mild cough that started 8 days ago and has progressed to include a worsening cough and increasing fatigue. His chest x-ray reveals bilateral hilar infiltrates and a CBC is normal. Which of the following diagnostic tests will most likely confirm the suspected diagnosis? A Acid-fast bacilli smear and culture B Blood culture C PCR testing of sputum D Sputum culture E Sputum gram stain

The Correct Answer is: C The patient most likely has atypical pneumonia secondary to mycoplasma pneumoniae which is best confirmed by PCR testing of sputum (C), oropharyngeal or nasal secretions and isn't detected through standard cultures (B, D) or staining techniques (A, E).

A 22 year-old male presents with a 2-week history of an upper respiratory infection that hasn't improved after taking amoxicillin for 6 days. He notes persistent sore throat, intermittent fever, and a worsening nonproductive cough. Physical examination reveals bilateral diffuse crackles. What is the most likely causative agent of this patient's symptoms? A Streptococcal pneumoniae B Klebsiella pneumoniae C Mycoplasma pneumoniae D Pneumocystis jiroveci E Staphlococcal aureus

The Correct Answer is: C The patient presentation is consistent with atypical pneumonia most commonly caused by viruses, mycoplasma pneumonia (C), chlamydia pneumonia, or legionella. The presentation of streptococcal pneumoniae (A) is more acute with productive cough and fever being early symptoms. Klebsiella pneumoniae (B) also presents with fever and a cough classically noted to be productive of "currant jelly" sputum. Pneumocystis jiroveci (D) causes pneumonia of insidious onset in immunocompromised patients. Staphlococcal (E) pneumonia also presents more acutely with productive cough.

Upon testing a patient for function of the hip abductors, which muscle is considered the primary muscle responsible for most abduction? A Biceps femoris B Gluteus maximus C Gluteus medius D Gluteus minimus E Vastus medialis

The Correct Answer is: C The primary mover in the motion of hip abduction is the gluteus medius muscle. Gluteus minimus does play a supporting role in that motion. Biceps femoris is one of the three hamstring muscles and contributes to the motions of knee flexion (primary muscle) and hip extension (secondary muscle). Gluteus maximus is the primary mover for hip extension and vastus medialis is one of the four quadriceps muscles responsible for knee extension, but no hip movements. (

A 14-year-old female patient presents to your family practice clinic having received a 1% total body surface area first and second degree burn to the left forearm. Of the following, what would you recommend for your patient? A Deroof any blisters, apply bacitracin topically, and prescribe pain medication, with follow-up in 48 hours, B Deroof any blisters, apply silver sulfadiazine topically, and prescribe pain medication, with follow-up in 48 hours. C Leave any blisters intact, apply bacitracin topically, and prescribe pain medication, with follow-up in 48 hours. D Leave any blisters intact, apply silver sulfadiazine topically, and prescribe pain medication, with follow-up in 48 hours, E Refer to the emergency department immediately,

The Correct Answer is: C Unless a critical surface (face, genitalia or hands) is involved, first and second degree burns may be treated in the outpatient setting. Blisters may be left intact as a physiologic dressing, and deroofed after they rupture. The patient requires tetanus prophylaxis and a topical antibiotic cream, usually either silver sulfadiazine or, preferably, bacitracin. Sulfadiazine may permanently stain skin, so use it cautiously in potentially exposed skin areas for cosmetic reasons.

A 49-year-old male presents to the clinic with symptoms of nausea, occasional vomiting, vague epigastric pain, fatigue, and weight loss of 35 lbs. over the past few months. On exam you find a palpable abdominal mass. Which of the following diagnostic tests is the best initial test to obtain? A Barium upper GI series B Abdominal CT C Upper endoscopy D PET scan E Flexible sigmoidoscopy

The Correct Answer is: C Upper endoscopy allows for a biopsy, which is highly sensitive in detecting gastric carcinoma, the suspected diagnosis. An upper GI series cannot distinguish a benign from a malignant lesion. An abdominal CT would be most useful once gastric carcinoma has been diagnosed, to help in preoperative evaluation. A PET scan would be most useful in detecting metastasis. A flexible sigmoidoscopy would not allow for visualization of the stomach.

What types of connective tissue are injured in a strain? A Bones and muscles B Fascia and joint capsules C Ligaments and joint capsules D Muscles and tendons E Tendons and bones

The Correct Answer is: D A strain involves injury to the muscles and tendons that are responsible for active movement of various body parts. Fascia is a part of the muscle-tendon unit, so injury to fascia would be considered a strain as well. Injury to ligaments and joint capsules would be considered a sprain and damage to bone would be classified as a fracture.

An avulsion fracture at the base of the fifth metatarsal is commonly called which of the following? A Bennett fracture B Boxer's fracture C Chauffer's fracture D Jones fracture E Lisfranc fracture

The Correct Answer is: D An avulsion fracture at the base of the fifth metatarsal, usually secondary to plantar flexion and inversion is called a Jones fracture. Also called a ballet or dancer's fracture, it is the most common metatarsal fracture. The fracture occurs at the proximal diaphysis. A Bennett fracture is an oblique fracture of the first metacarpal near the carpometacarpal joint. A boxer's fracture is a fracture of the fifth metacarpal. This is the most common fracture of the hand. A chauffer's fracture is an oblique fracture through the base of the radial styloid in the forearm. A Lisfranc fracture is actually a fracture and dislocation involving the tarsometatarsal joints.

An 8-month-old male presents to the clinic due to irritability, fatigue and parental concerns about developmental delays. He is fifth percentile for weight and 33rd percentile for height. An office based hemoglobin level is 8.4 mg/dL. The infant is prescribed iron supplementation, to begin today. Which of the following tests should be performed in 5-7 days to confirm response and adherence to iron supplementation? A Ferritin B Hemoglobin C Hematocrit D Reticulocyte count E Serum iron

The Correct Answer is: D An elevated reticulocyte count 1 week after initiation of iron supplementation confirms the presence of IDA and efficacy of therapy.

A 16-year-old male soccer player is complaining of pain to the right foot that has been getting progressively worse for the last 2 months. He states it hurts the most when he has all of his weight on his right foot as he plants to kick the ball. Most of the pain appears to be on weight bearing. You are concerned that this patient may be developing a stress fracture. Based on the patient's history and patient presentations, which bone is the most affected by stress fractures in the foot? A Calcaneus B Fifth metatarsal C First metatarsal D Second metatarsal E Talus

The Correct Answer is: D Any bone that is exposed to repetitive stress can have a stress fracture, but the long and thin metatarsal bones of the foot are the most commonly affected bones. Of the metatarsals, the second metatarsal has the highest number of stress fractures. These weight bearing bones can be particularly vulnerable to stress fracture if the patient is involved in long distance running, especially if he/she is wearing improper footwear for that activity or footwear that has lost most of its shock absorbing abilities. Some young female athletes may be training so hard that they become amenorrheic which can contribute to osteopenia resulting in weaker bones. Older patients with osteoporosis will also have a higher risk of stress fracture. Initially stress fractures of the metatarsals may present with a small area of localized pain and the dorsal forefoot may demonstrate a fairly diffuse area of swelling. If the stress fracture is not treated early, some patients will experience an audible pop or crack as the incomplete stress fractures progresses to a complete break. All types of fractures occur more easily in long thin bones like the metatarsals, than thicker bones like the calcaneus and talus

What nerve is most commonly injured in a mid- or distal humeral shaft fracture? A Axillary B Median C Peroneal D Radial E Ulnar

The Correct Answer is: D Because of the radial nerves proximity to the humerus, mid and distal shaft fractures with significant displacement can cause a radial nerve injury. Median and ulnar injuries are more commonly associated with forearm injuries. Axillary nerve injuries are most common in anterior shoulder dislocations and peroneal nerve damage occurs as a result of lower leg insult.

Your 36-year-old obese female patient complained of a single episode of right upper quadrant pain after eating fast food (a double cheeseburger, fries, and a vanilla shake) last week. The symptoms have resolved and have not reoccurred when she presents to your family practice. Your psychical examination is completely normal, as is blood work, including a complete blood count, comprehensive metabolic panel, acute viral hepatitis panel, amylase, and lipase. You suspect cholecystitis and order an abdominal sonogram. The radiologist makes note that the gall bladder appears thickened with an associated calcification with a single, large gallstone. Of the following, what is the most appropriate next step? A Counsel the patient on diet, exercise, and lifestyle changes and revisit the issue should symptoms reoccur. B Refer the patient for an abdominal computed tomography (CT) scan. C Refer the patient to a gastroenterologist. D Refer the patient to a surgeon for a laparoscopic cholecystectomy due to the risk of gall bladder cancer. E Re-evaluate the patient for symptomatology in four to six weeks.

The Correct Answer is: D Calcification of the gall bladder, frequently referred to as a porcelain gall bladder, and single, solitary, large, gall stone are a common presentation of gall bladder cancer on imaging. Especially in the face of a symptomatic patient, cholecystectomy with an associated biopsy should be a strong consideration.

A 41-year-old alcoholic male, who lives primarily on the streets, appears pale, cachectic, and mildly icteric. He is complaining of several weeks of increasing fatigue. Laboratory findings note an elevated MCV of 128. What other physical finding would most support the diagnosis for megaloblastic anemia from folate deficiency? A Decreased vibration and position sense B Dementia C Difficulty with balance D Glossitis E Parethesias

The Correct Answer is: D Features of folate deficiency are similar to vitamin B12 deficiency. However, there are none of the neurologic abnormalities associated with vitamin B12. Glossitis is the only non-neurologic finding in the PE that would support folate deficiency. Alcoholism and poor dietary intake also support the diagnosis of folate deficiency.

A 28-year-old patient who is a fire department paramedic presents for a routine physical examination to your family practice office. They are asymptomatic, but their PPD is positive. Suddenly, they relay that they have tested positive on their PPD in the past, "about five years ago," and that they were treated at that time with nine months of INH. What should your next step be in treating them? A Begin treatment for resistant tuberculosis (TB). B Nothing, inasmuch as they have already received a full course of therapy. C Order a chest x-ray and if normal, follow the patient with chest x-rays every two to three years or sooner if they become symptomatic D Order a chest x-ray, and if normal, clear the patient cautioning them to immediately contact you should they develop any signs or symptoms consistent with TB. E Refer the patient to ID or Pulmonary for definitive treatment of resistant TB.

The Correct Answer is: D Health care workers (HCWs) with positive PPD test results should have a chest radiograph as part of the initial evaluation of their PPD test; if negative, repeat chest radiographs are not needed unless symptoms develop that could be attributed to TB. However, more frequent monitoring for symptoms of TB may be considered for recent converters and other PPD-positive HCWs who are at increased risk for developing active TB (e.g., HIV-infected or otherwise severely immunocompromised HCWs). Regardless of whether the patient completes treatment for latent TB infection, serial or repeat chest radiographs are not indicated unless the patient develops signs or symptoms suggestive of TB disease.

An 8-month-old male presents to the clinic due to irritability, fatigue and parental concerns about developmental delays. He is fifth percentile for weight and 33rd percentile for height. An office based hemoglobin level is 8.4 mg/dL. Which of the following is the most likely diagnosis? A Alpha thalassemia minor B Beta thalassemia minor C Folate deficiency D Iron deficiency anemia E Sideroblastic anemia

The Correct Answer is: D Infants with poor nutrition are at greatest risk of iron deficiency anemia between the ages of 6 and 24 months. Alpha thalassemia minor (A) and beta thalassemia minor (B) present with a markedly microcytic, but mild anemia. Folate deficiency (C) due to inadequate nutrition in the absence of co-morbid disease is rare. Sideroblastic anemia (E) is far less common than iron deficiency anemia and typically diagnosed based on the finding of ringed sideroblasts visible on peripheral smear.

A 19 year-old woman presents to the clinic complaining of fever, and productive cough. Physical exam reveals a temperature of 102F, decreased breath sounds of her right middle lobe. She is prescribed levofloxacin. What is the mechanism of action of levofloxacin? A Binds to the 30S ribosomal subunit to inhibit protein synthesis B Binds to the 50S ribosomal subunit to inhibit protein synthesis C Inhibits dihydropteroate synthase and folate production D Inhibits DNA replication by binding to DNA gyrase and topoisomerase IV E Inhibits the transpeptidation reaction

The Correct Answer is: D Levofloxacin inhibits DNA replication by binding to DNA gyrase and topoisomerase IV (D). Tetracycline binds to the 30S ribosomal subunit to inhibit protein synthesis (A). Macrolide antibiotics bind to the 50S ribosomal subunit to inhibit protein synthesis (B). Sulfamethoxazole inhibits dihydropteroate synthase and folate production (C). Beta lactam antibiotics inhibit the transpeptidation reaction leading to cell wall destruction (E).

A 68-year-old female with a past medical history of diabetes, dyslipidemia, hypertension, and gout presents with a gradual progression of fatigue and pallor over the last few months. Initial CBC results show a hemoglobin of 10.4 mg/dL, hematocrit of 32%, an MCV of 112 fL, and a reticulocyte count of 0.1%. Which of the following medications should be considered as a potential cause of her anemia? A Allopurinol B Atorvastatin C Lisinopril D Metformin E Nicotinic acid

The Correct Answer is: D Metformin is associated with vitamin B 12 deficiency and may rarely be a medication cause for macrocytic anemia. Nicotinic acid (E) administration may help reduce the risk of vitamin B 12 deficiency secondary to nicotinic acid deficiency. Allopurinol (A), atorvastatin (B), and lisinopril (C) are not strongly associated with vitamin B 12 or folate deficiency.

A 33 year-old man with cystic fibrosis is admitted to the hospital with pneumonia. You order a sputum culture and sensitivity. What is the most likely organism causing this patient's pneumonia? A Klebsiella pneumoniae B Legionella C Mycoplasma pneumoniae D Pseudomonas aeruginosa E Streptococcus pneumoniae

The Correct Answer is: D Patients with Cystic Fibrosis commonly develop chronic infection and subsequent pneumonia from

A 56-year-old male who works in construction climbing ladders has developed pain to the right foot for several days. You have seen and examined the patient a few days after the patient started complaining of pain to the foot. Your initial x-rays of the foot are negative for fracture. He continues to have pain, and decreased ability to bear weight. Based on this clinical scenario, how many days after the initial examination should another x-ray be ordered to look for a stress fracture? A 1-2 days B 5-7 days C 10-12 days D 14-30 days E 180 days

The Correct Answer is: D Radiographic evidence of stress fractures is not immediately apparent after the initial onset of symptoms. Estimates of anywhere from 14 to 42 days before visible signs of a stress fracture can be detected on X-ray have been put forth with a commonly utilized range of 14 to 30 days. Any answer choice above that included numbers less than 14 days would simply be inaccurate due to the gradual appearance of the classic x-ray findings of a stress fracture. After 180 days from the time of stress fracture onset, the fracture should be nearly healed if managed properly. If a stress fracture is highly suspected in light of negative x-rays, imaging with a bone scan should help make the diagnosis. MRI scans may help confirm the diagnosis, but they are rarely utilized in the work-up of suspected stress fractures

A 72-year-old female presents with a several month history of symmetric pain to the hands, wrists, and feet. She also is noticing a change to the appearance of the joints of the hands and feet, with a more "nobby" appearance than before. When the patient wakes up in the morning, the joints are particularly stiff and red at times. Out of all the areas that are painful, it is the hands that are the most painful. Based on this history, what physical exam findings would you expect to find on the hands that are classic findings of this disease? A Radial deviation and subluxation of the DIP joints B Radial deviation and subluxation of the MCP joints C Radial deviation and subluxation at the PIP joints D Ulnar deviation and subluxation at the MCP joints E Ulnar deviation and subluxation at the PIP joints

The Correct Answer is: D Rheumatoid arthritis is a systemic autoimmune disease that is incurable and affects woman more than men by a 3 to one ratio. It can damage any joint, but it most often affects joints of the hands, wrists, knees, feet and ankles. The classic appearance of hands that have been afflicted with progressive rheumatoid arthritis is that they demonstrate ulnar deviation and subluxation at the MCP joints. Involvement of the PIP joints is not uncommon, but DIP joints are generally not as significantly affected. Radial deviation is not a typical characteristic of rheumatoid arthritis of the hands. (

A 42-year-old woman presents complaining of shortness of breath, three days of fever as high as 103F, and a cough productive of green sputum. On physical examination, you hear crackles in her lungs. A chest radiograph reveals a consolidation in the left lower lobe. What tactile fremitus findings do you expect to observe in the left lower lobe region? A Absent B Decreased C Hyperresonant D Increased E Normal

The Correct Answer is: D Tactile fremitus is increased (D) by processes that cause consolidation within the lung parenchyma and decreased (B) by processes that insulate the lung parenchyma from transmitting vibration (e.g., pneumothorax, hemothorax) or airway obstruction.

A 7-year-old girl is brought in by her mom for evaluation of a rash. She has had a fever for a few days and woke up this morning looking like she had been slapped on both cheeks. Other than supportive care, which instruction below represents the best patient education for this patient? A She should remain out of school because she is contagious until the rash resolves. B She may return to school but stay out of physical education class to avoid splenic injury. C It spreads by the fecal-oral route, so she should wash her hands after using the bathroom. D She may resume normal activities as her energy level improves.

The Correct Answer is: D The "slapped cheek" appearance to the rash is consistent with a Parvovirus B19 etiology for erythema infectiosum. It is a droplet infection that is no longer contagious once the rash breaks out. It generally has a benign course and patients recover fully with supportive care. Splenic involvement is not typically a part of the course, so she may resume activities as she feels able.

A 5-year-old boy presents to urgent care complaining of painful lesions in his mouth that have made eating difficult the past 2 days. The mother confirms he has been unable to eat for 48 hours, but has been able to sip water. On physical exam he has a temperature of 102.6 F; numerous small vesicles and ulcers on the buccal mucosa and tongue, inflamed gingiva; and tender anterior cervical adenopathy. Which of the following is the most likely diagnosis? A Aphthous ulcers B Hand, foot, and mouth disease C Herpangina D Herpes simplex gingivostomatitis E Viral pharyngitis

The Correct Answer is: D The classic presentation of initial herpes simplex infection (D) includes multiple small, painful vesicles or ulcers on the mucousa with gingival involvement, fever, and adenopathy. (A), (B), and (C) all present with ulcers, but typically involve an isolated area (A), or the tonsils and posterior pharynx (B and C).

A 5-year-old boy presents to urgent care complaining of painful lesions in his mouth that have made eating difficult the past 2 days. The mother confirms he has been unable to eat for 48 hours, but has been able to sip water. On physical exam he has a temperature of 102.6º F; numerous small vesicles and ulcers on the buccal mucosa and tongue; inflamed gingiva; and tender anterior cervical adenopathy. Which of the following is the most likely causative organism? A Coronavirus B Coxsackie virus A16 C Group A beta-hemolytic strep D Herpes simplex 1 E Rhinovirus

The Correct Answer is: D The classic presentation of initial infection with herpes simplex virus 1 (D) includes multiple small, painful vesicles or ulcers on the mucousa with gingival involvement, fever, and adenopathy. Coxsakie virus A16 (B) causes hand, foot, and mouth disease. Coronavirus (A) and rhinovirus (E) cause viral pharyngitis.

A 26 year-old man presents to the clinic after developing a severe productive cough, fever, chills, night sweats and weight loss. Which is the most definitive diagnostic test for determining the presence of Mycobacterium tuberculosis as the causative agent of his active pulmonary tuberculosis and yields results in less than 24 hours? A Acid-fast bacilli smear of sputum B Blood culture C Mycobacterial culture of sputum D Nucleic acid amplification of sputum E Tuberculin skin test

The Correct Answer is: D The most definitive diagnostic tests for confirmation of tuberculosis are mycobacterial culture (C) which requires at least 2 weeks to grow mycobacterium, and nucleic acid amplification (D) of a respiratory specimen that can be completed in a matter of hours. AFB smear (A) has a sensitivity of only 40-60%, but positive results help establish a presumptive diagnosis. Tuberculin skin testing (D) is effective in screening patients for latent TB. Routine blood cultures (B) are ineffective in diagnosing TB.

A 48-year-old man presents to the clinic for a routine employment physical. The patient is asymptomatic. A pre-employment CBC reveals hemoglobin of 13.2g/dl, hematocrit of 39.5%, and MCV of 60.6 fL. Subsequent iron studies, hemoglobin electrophoresis, and sickle cell screening are "normal." After the labs are reviewed, the patient states "they are always normal." Which of the following is the most appropriate management for this patient? A Bone marrow aspirate B Ferrous sulfate daily C Folic acid supplements D Reassurance E Refer to an oncologist

The Correct Answer is: D The patient has a mild anemia with pronounced microcytosis consistent with alpha thalassemia minor and requires reassurance. Alpha thalassemia minor hemoglobin electrophoresis reveals normal results. Patients with alpha thalassemia are often inappropriately advised to take iron supplements (B) and don't require iron, folic acid (C) or vitamin B 12 supplements unless a coexisting deficiency is confirmed. Referral to an oncologist (E) and a bone marrow aspirate (A) aren't necessary for this patient.

A 48-year-old female presents with a gradual progression of fatigue, pallor, and dyspnea on exertion over the last few months. Initial CBC results show a hemoglobin of 10.2 mg/dL, hematocrit of 31%, an MCV of 74 fL, and a reticulocyte count of 0.1%. Which of the following is the most likely pathophysiologic mechanism responsible for her anemia? A Acute blood loss B Defective bone marrow/stem cell function C Defective DNA production D Defective hemoglobin production E Increased destruction of red blood cells

The Correct Answer is: D The patient most likely has an underlying iron deficiency anemia (IDA), which requires confirmation and then evaluation for sources of chronic blood loss. IDA results from defective hemoglobin production.

A 23-year-old man presents to the outpatient clinic for follow-up from a recent urgent care visit. He complains of sore throat, fever, fatigue, myalgias, and a rash that started 5 days ago, and have worsened since he was seen in the urgent care 3 days ago. The patient appears non-toxic with a temperature of 39.4 degrees Celsius. Physical exam reveals pharyngeal and tonsillar erythema without exudates, generalized lymphadenopathy, a morbilliform rash on his trunk, and no hepatosplenomegaly. A rapid strep screen and Monospot performed at the local urgent care were reportedly negative. Which of the following laboratory tests is most likely to confirm the expected diagnosis? A Complete blood count B Cytomegalovirus titer C Group A beta-hemolytic strep culture D HIV viral load E Epstein-Barr virus titer

The Correct Answer is: D The patient presentation is consistent with acute retroviral syndrome, which is best confirmed during this initial presentation phase through direct testing for the HIV virus, such as an HIV viral load (D). The lack of tonsillar exudates, a negative monospot, and presence of generalized adenopathy make infectious mononucleosis (B and E) less likely. A CBC (A) may show lymphopenia and support the diagnosis, but it doesn't confirm the diagnosis.

A 45-year-old man presents to the clinic complaining of a lesion in the middle of his right lower eyelid margin that is obscuring his vision when he works on fly-tying or other detailed projects. He has been trying conservative treatment with warm compresses for the past several weeks. On physical exam there is a 5 mm round lesion that lacks erythema or tenderness. The patient's visual acuity is 20/20. Which of the following interventions is most appropriate for this patient? A Apply warm compresses four times daily until resolution B Apply bacitracin ophthalmic ointment 500 units/gram three times a day C Aspirate the lesion D Excise the lesion E Take amoxicillin-clavulanate 875/125 mg by mouth two times a day for 10 days

The Correct Answer is: D The patient's presentation is consistent with a chalazion that is best treated by surgical excision of the granulomatous tissue (D). Warm compresses are appropriate treatment initially for 10-15 minutes for about 3-4 times a day. Since this patient has been trying this treatment for the past several weeks the next line of therapy would be excision (D). Warm compresses (A), bacitracin ointment (B), and amoxicillin-clavulanate (E) are effective treatment options for hordeolum, or hordeoulum with associated preseptal cellulitis.

A 12-year-old boy is being seen for concerns of development of breast tissue. Upon physical exam, he is noted to have a firm, slightly tender mass under the left areola. What is the most appropriate action at this time? A referral to pediatric surgery for resection B measurement of serum hCG C measurement of testosterone and estrogen levels D reassurance and observation

The Correct Answer is: D Type 1 idiopathic gynecomastia in adolescent men presents with a firm mass under the areola ("breast bud") typically during sexual maturation stages (SMR), stages II to III. This is a result of normal estrogen and androgen activity at the breast tissue level. Appropriate action is observation and to reassure the patient that the condition will likely resolve in 1 to 2 years. (Kaplan, 2009, p. 120) Kaplan DW , Love-Osborne KA. Adolescence. In: Hay WW , Levin MJ , Sondheimer JM, et al., eds. Current Pediatric Diagnosis and Treatment. 19th ed. New York, NY: McGraw-Hill; 2009.

A 71-year-old male who has a history of hypertension presents with a new finding of atrial fibrillation. He is independent, drives his own car, and tends to his daily activities without assistance. He is currently having no symptoms, and his heart rate is 90, with a blood pressure of 146/76. Given this clinical scenario, what is the best pharmacologic anticoagulation treatment for this patient? A No anticoagulation B Aspirin C Aspirin and warfarin D Aspirin and clopidogrel E Warfarin

The Correct Answer is: E This patient has a CHA2DS2-VaSc score of 2 (HTN and age) and would have adequate risk reduction with the therapy of anticoagulation such as warfarin. Higher risk patients with a CHADS2 score of 2 or higher qualify for adding warfarin (C), and the use of aspirin and clopidogrel (D) is not indicated.

A 5 year-old girl presents to the clinic with her mother who is concerned about her daughter's energy level since starting full day kindergarten. CBC reveals a hemoglobin of 12.3 g/dl, hematocrit of 36%, MCV of 62 fL, and an MCHC of 34 g/dL. Serum ferritin levels are normal. What is the most likely hemoglobin electrophoresis results for this patient? A Decreased HbA2 and increased HbF levels B Decreased HbA2 and HbF levels C Increased HbA2 and normal HbF levels D Increased HbA2 and HbF levels E Normal HbA2 and HbF levels

The Correct Answer is: E Alpha thalassemia presents with a normal hemoglobin electrophoresis, since all adult hemoglobin are alpha-containing and affected equally. Beta thalassemia major may present with increased HbA 2 and/or HbF levels (C and D). Beta thalassemia minor typically presents with increased HbA 2 (C).

A fracture involving the medial epicondyle will most likely cause damage to which nerve? A Axillary B Median C Peroneal D Radial E Ulnar

The Correct Answer is: E Because the ulnar nerve passes through the cubital tunnel, which is a groove on the posterior aspect of the medial epicondyle, any fractures involving the medial epicondyle can also cause damage to the ulnar nerve. The median nerve is most susceptible to injury at the carpal tunnel. Branches of the radial nerve can become entrapped on the lateral side of the elbow and the associated symptoms are often confused with lateral epicondylitis. Radial nerve injuries are more commonly associated with humeral shaft fractures. The axillary nerve is significantly proximal to the medial epicondyle and the peroneal nerve is in the leg.

A 2 year-old girl presents to the clinic due to fever, and right ear pain. Physical exam reveals a temperature of 103.1 F and an erythematous and immobile right tympanic membrane. The patient is subsequently prescribed amoxicllin. What is the mechanism of amoxicillin? A Binds to the 30S ribosomal subunit to inhibit protein synthesis B Binds to the 50S ribosomal subunit to inhibit protein synthesis C Inhibits dihydropteroate synthase and folate production D Inhibits DNA replication by binding to DNA gyrase and topoisomerase IV E Inhibits the transpeptidation reaction

The Correct Answer is: E Beta lactam antibiotics inhibit the transpeptidation reaction leading to cell wall destruction (E). Tetracycline binds to the 30S ribosomal subunit to inhibit protein synthesis (A). Macrolide antibiotics bind to the 50S ribosomal subunit to inhibit protein synthesis (B). Sulfamethoxazole inhibits dihydropteroate synthase and folate production (C). Ciprofloxacin inhibits DNA replication by binding to DNA gyrase and topoisomerase IV (D).

A 73 year-old woman with a past medical history of diabetes and hypertension presents to the clinic complaining of progressive fatigue. An initial CBC reveals a hemoglobin 10.1 mg/dL and MCV of 91 fL. Which of the following best describes the cause of the abnormality pictured in the patient's peripheral smear? (note photo taken from figure 57-5- in Harrison's) A Absent or non-functioning spleen B Failure of nuclear maturation C Intravascular hemolysis D Lead intoxication E Presence of uremia

The Correct Answer is: E Burr cells are present on the peripheral smear and are associated with uremia.

A 21-year-old female patient presents with a large area of swelling localized to the front of the knee, between the patella and the skin after a fall directly on her knee. She has been stating that her knee is becoming more difficult to move due to the swelling and pain. She is otherwise healthy with no other medical problems. Her exam reveals a tender, fluctuant area just anterior to the patella without warmth. She does not appear to have any effusion, and her ligament stability testing is normal. Based on these findings, what is the most likely diagnosis? A Anterior cruciate ligament tear B Medial meniscal tear C Pes anserine bursitis D Posterior cruciate ligament tear E Prepatellar bursitis

The Correct Answer is: E Bursae are synovial fluid filled sacs that facilitate the reduction of friction between adjacent structures. They may be found between the skin and various bony prominences or between tendons, ligaments and bone. They can become injured by an acute direct impact or gradual repetitive stress such as what might occur if someone was required to do extensive kneeling on the job. They may also be sites of infection with staphylococcus aureus and streptococcus species being the most common pathogens. The knee contains two primary bursae. The prepatellar bursa lies directly below the skin and above the patella. It is often acutely injured with a fall or other traumatic impact. A localized swelling can occur quickly and this fluid accumulation is not intracapsular as occurs with a joint effusion after an anterior cruciate ligament tear, meniscal tear or posterior cruciate tear. Meniscal injuries tend to produce a smaller effusion than cruciate ligament tears. The mechanism of falling on a bent knee can be consistent with a posterior cruciate ligament injury, but given the lack of an effusion and the presence of a well contained anterior knee swelling, this becomes less likely. Pes anserine bursa are located on the anterior medial aspect of the tibia, just below the tibial plateau. This location is the insertion of the gracilis, sartorius, and semitendinosus muscles. Pes anserine bursitis is commonly associated with early osteoarthritis in the medial compartment of the knee.

A 57-year-old woman with a history of rheumatic fever is seen complaining of dyspnea while vacuuming her apartment, which has been worsening over the last few months. On physical exam, a possible opening snap, loud S 1 , and a very soft diastolic rumbling murmur is auscultated. When the patient is placed in the left lateral decubitus position, the murmur is accentuated, and heard best at the apex. With inspiration, the murmur does not increase in amplitude. Which of the following is the most appropriate next diagnostic study? A Chest x-ray B Transesophageal echocardiogram C Holter monitor D Treadmill exercise stress test E Transthoracic echocardiogram

The Correct Answer is: E Choice E, transthoracic echocardiogram, is a simple, sensitive and non-invasive diagnostic tool, which can evaluate for the presence of valvulopathy in the setting of a patient with a diastolic murmur and a history of rheumatic fever. Choice B is also useful for an evaluation for the presence of valvulopathy, but is more invasive than choice E; therefore, choice E is the more appropriate study. Choice A might be able to give evidence of cardiomegaly, but would not be sensitive enough to detect valvulopathy. Choice C is a useful diagnostic tool for the evaluation of patients complaining of palpitations, but incorrect for this patient who has no symptoms. Choice D, although a useful diagnostic tool for the evaluation of exercise tolerance and in patients complaining of chest pain, does not allow direct visualization of the heart valves to evaluate for valvulopathy.

Approximately what percentage of patients with a solid primary tumor elsewhere will end up with metastatic disease of the vertebrae during the clinical course of their cancer? A 10% B 20% C 30% D 40% E 50%

The Correct Answer is: E Fifty percent of cancer patients will develop metastatic disease of the vertebrae at some point during the course of their illness. The highest percentages of cancers that lead to such spinal lesions are carcinomas of the breast, lung, prostate, colon, thyroid and kidney. This likely occurs through hematogenous spread. Pain is a common presenting symptom, but it may be found while still asymptomatic if routine screenings are done with bone scans, MRI or CT in patients with a known primary tumor elsewhere in the body.

A 64-year-old female who has a history of injectable drug use presents with blood work that reveals leukocytosis with a left shift, and there is suspicion of osteomyelitis based on the patient's prior history. Based on this history, what bone would be most affected by hematogenous osteomyelitis in adults? A Feet B Long bones C Pelvis D Sternoclavicular bones E Vertebrae

The Correct Answer is: E Hematogenous osteomyelitis accounts for about 20% of all cases of osteomyelitis in adults. It is more common in males and the prevalence is higher amongst those who are IV drug abusers, patients being treated with dialysis or who have sickle cell disease. Other conditions which may lead to sepsis (i.e. patients with central lines, urinary infections, and urethral catheterization) increase the risk of hematogenous osteomyelitis. Unlike children, the long bones are rarely affected in adults with the vertebrae being the most likely location for the bone infection to occur. Lumbar vertebrae are most often affected, followed by thoracic and cervical vertebrae.

A 67 year-old man with chronic obstructive pulmonary disease is prescribed ipratropium bromide inhaler. Which of the following is a potential side effect associated with ipratropium? A Bradycardia B Diarrhea C Salivation D Sweating E Urinary retention

The Correct Answer is: E Ipratropium is an anticholinergic agent that can directly cause tachycardia, ileus, dry mouth, flushed/warm/dry skin, delirium, and urinary retention (E). Answer choices A, B, C and D result from cholinergic agents

A 15-year-old male presents complaining of a sore throat, headache, and mild cough that started 8 days ago and has progressed to include a worsening cough and increasing fatigue. His chest x-ray reveals bilateral hilar infiltrates, CBC is normal and a nasal secretions test positive for mycoplasma pneumoniae by PCR. What is the most appropriate therapy? A Amoxicillin B Cefuroxime C Clindamycin D Ciprofloxacin E Doxycycline

The Correct Answer is: E Mycoplasma pneumoniae is commonly treated with macrolides, doxycycline (E), or respiratory fluoroquinolones. Mycoplasma pneumonia doesn't respond to beta-lactam antibiotics (A, B) or non-respiratory fluoroquinolones (D).

A 38 year-old woman with severe-persistent asthma presents to the clinic complaining of nightly nighttime awakens and frequent use of her inhaled albuterol. Which of the following is an antibody that prevents IgE from binding to its receptor on mast cells and basophils and may be considered as a treatment for this patient? A adlimumab B daclizumab C etanercept D infliximab E omalizumab

The Correct Answer is: E Omalizumab (E), is an antibody that prevents IgE from binding to its receptor on mast cells and is used in the treatment of allergic disease. Daclizumab (B) is anti-IL-2 antibody used to prevent acute rejection of organ transplants. Adlimumab (A), etanercept (C), and infliximab (D) bind TNF, thus inhibiting the action of TNF and are used in the treatment of disorders such as psoriasis and rheumatoid arthritis.

A 56-year-old insulin dependent diabetic has been under your evaluation for his diabetes for several years. The patient has a 3-year history of diabetic neuropathy to the right foot, and may have suffered an injury to the foot without knowing due to loss of sensation. The patient now presents with a tender, reddened, and swollen right foot for the last 10 days that is also warm to the touch. You suspect that this patient may have an acute case of osteomyelitis. Based on this history, what bacterial organism is most commonly the cause of osteomyelitis? A Group A beta-hemolytic streptococci B Hemophilus influenzae C Mycoplasma D Pseudomonas aeruginosa E Staphylococcus aureus

The Correct Answer is: E Osteomyelitis is an infection in a bone and can occur in patients of all ages. The most common organism implicated in osteomyelitis across all age groups is Staphylococcus aureus. It can enter the bone through multiple mechanisms including by direct inoculation during an open fracture or during surgical intervention following a fracture (most common mechanisms for adults) or by hematogenous spread from another source (the usual cause in children). Hemophilus influenzae was a much more common organism in pediatric osteomyelitis in the past, but its prevalence is decreasing due to routine immunizations. Group A beta-hemolytic streptococci is the second most common organism found in osteomyelitis in children, while Pseudomonas aeruginosa is the second most common pathogen in adults. Mycoplasma induced osteomyelitis is relatively rare and usually confined to immunocompromised patients

A 22-year-old patient with sickle cell disease presents to the emergency department complaining of chest pain, fever, and non-productive cough. On physical exam his temperature is 100.6˚F, BP is 144/88, pulse is 110, respiratory rate is 24, and pulse oximetry is 84%. CBC shows a WBC of 11,500, hemoglobin of 8.3%, and hematocrit of 28%. What is the most critical/emergent treatment for this patient? A Acetaminophen B Bone marrow transplant C Furosemide D Hydroxyurea E Transfusion

The Correct Answer is: E Patients with sickle cell disease are prone to acute chest syndrome resulting from sickling of cells within the lung that typically presents with chest pain, tachypnea, cough, fever, and oxygen desaturation. Transfusion is indicated when the hematocrit drops below 30% or oxygen saturation drops below 90%.

A 64-year-old, nonsmoking male presents to the clinic with complaints of headache and dizziness. Initial labs reveal hemoglobin of 20.1 g/dl, hematocrit of 60%, and platelet count of 567,000. Which of the following is the most likely physical exam finding in this patient? A Erythromelalgia B Hepatomegaly C Loss of vibratory sense D Petechiae E Splenomegaly

The Correct Answer is: E Splenomegaly is the most common physical exam finding in patients with polycythemia vera (PCV) and may be accompanied by hepatomegaly (B). Erythromelalgia (A) also occurs in patients with PCV due to increased red blood cell mass, but occurs less commonly than splenomegaly. Patients with PCV often have thrombocytosis, which most commonly contributes to thrombotic events, but less commonly may result in abnormal platelet function and petechiae (D) and increased bleeding risk. PCV is associated with elevated B 12 levels and not likely to result in the classic neurologic findings of B 12 deficiency including peripheral neuropathy assessed by vibratory sensation (C), ataxia and dementia.

A 16-year-old male high school wrestler presents to your family practice clinic with a fluctuant 3 cm by 3 cm abscess to his back superior and lateral to his scapula. Which of the following is the most appropriate treatment? A amoxicillin-clavulanate orally for 10 days B cephalexin orally for 10 days C trimethoprim sulfamethoxazole orally for 10 days D vancomycin intravenously for 10 days E incision and drainage is likely to resolve the abscess without the need for medications

The Correct Answer is: E The causative agent of this abscess is most likely caused by community-acquired methicillin-resistant S aureus (caMRSA). Infectious Diseases Society of America (IDSA) guidelines issued in January of 2011 generally recommend incision and drainage alone for fluctuant abscesses in an otherwise immunocompetent patient.

A 64-year-old, nonsmoking male presents to the clinic with complaints of headache and dizziness. Initial labs reveal hemoglobin of 20.1 g/dl, hematocrit of 60%, and platelet count of 567,000. Which of the following laboratory finding confirms the suspected diagnosis? A Bone marrow hypocellularity B Elevated ferritin C Elevated serum erythropoietin D Presence of Philadelphia chromosome E Presence of JAK2 mutation

The Correct Answer is: E The clinical presentation is most consistent with polycythemia vera (PCV), which, in the presence of a markedly elevated red blood cell mass, is confirmed by the presence of the JAK2 mutation. The patient's erythropoietin (C) and ferritin (B) are most likely decreased as a result of bone marrow overproduction of RBCs and use of iron stores to produce these RBCs. The bone marrow of patients with PCV is typically hypercellular (A) across all cell lines.

A 24-year-old woman comes to your office complaining of episodic chest pains that have been occurring over the past several months. She experiences shortness of breath with each occurrence and feels like "I'm going to die." She denies cough, fever, or sputum production and has noticed the episodes occur when she is in a crowd. Her current examination is normal and she feels well between episodes. You suspect panic disorder, but want to rule out the most likely organic causes. Which of the following tests will you include in your initial work-up? A Arterial blood gases B CBC C Chest x-ray D Sputum culture E Urine toxicology screen

The Correct Answer is: E The differential diagnosis of panic disorder includes many cardiac, pulmonary, and endocrine conditions, but in a young woman without any symptoms or signs between episodes, drug use, as detected on a urine toxicology screen (E), is the most likely cause. An EKG might also be considered to rule out a cardiac cause. CBC (B) and sputum culture (D) might be helpful if infection is a consideration, but she has no fever or other signs of infection. Chest x-ray (C) and blood gases (A) would not be helpful without symptoms or signs suggesting particular etiologies.

What is the first type of motion that is lost as a person develops progressively worsening osteoarthritis of the hip joint? A Abduction B Adduction C External rotation D Flexion E Internal rotation

The Correct Answer is: E The earliest sign of the development of osteoarthritis of the hip is often the loss of internal rotation. As the condition worsens, muscular contractures may develop which hold the affected limb in a flexed and externally rotated position, which has great consequence on the individual's gait and functioning level. Typically a person experiencing this type of osteoarthritis will eventually develop an antalgic gait where the time spent bearing weight on the affected limb is brief due to pain. Also, the gluteus medius (which is a hip abductor and helps stabilize the pelvis) may become weakened as the condition worsens, resulting in an abductor lurch as the trunk of the body sways out over the affected limb when attempting to walk.

A 48-year-old female presents with a gradual progression of fatigue, pallor, and dyspnea on exertion over the last few months. Initial CBC results show a hemoglobin of 10.2 mg/dL, hematocrit of 31%, an MCV of 74 fL, and a reticulocyte count of 0.1%. After treatment for her anemia begins, which of the following diagnostic studies can be ordered first (earliest) to demonstrate a response to therapy? A Bone marrow aspirate B Ferritin C Hemoglobin/hematocrit D Iron binding capacity E Reticulocyte count

The Correct Answer is: E The patient most likely has an underlying iron deficiency anemia (IDA) which requires confirmation and then evaluation for sources of chronic blood loss. IDA is treated with iron replacement (e.g., ferrous sulfate) and response to therapy can be assessed in 7-10 days via a reticulocyte count. The elevation of the reticulocyte count (after initial reticulocyte count was relatively low) at this time demonstrates the return of normal hemoglobin production (E). Bone marrow aspirate (A) would be an inappropriately invasive test to perform in this scenario. The patient's hemoglobin, hematocrit (C), MCV, and iron studies (B and D) will take a couple to a few months to return to normal.

A 36 year-old woman with no significant past medical history presents with gradual onset of dyspnea and fatigue leading to an episode of "fainting" this morning. Physical exam reveals increased jugular venous pressure, weak carotid pulses, clear lungs, and a loud S2. What is the most likely diagnosis? A Aortic stenosis B Cardiac tamponade C Mitral Regurgitation D Pulmonary fibrosis E Pulmonary hypertension

The Correct Answer is: E The patients symptoms are due to decreased cardiac output resulting from decreased preload associated with pulmonary hypertension (E). Aortic stenosis (A) presents more commonly in geriatric patients who present with a murmur. Cardiac tamponade (B) can decrease cardiac output, but would lead to decreased heart sounds. Mitral regurgitation (C) would cause pulmonary edema and rales in conjunction with increased jugular venous pressure. Pulmonary fibrosis (E) is unlikely in this patient with normal lung sounds.

You are evaluating a patient who is having decreased sensation to his arm after he sustained a head on injury while playing football. He is stating that his right arm has decreased sensation that goes into the hand, but he is slightly vague on being descriptive. As you perform the physical examination, what part of the body would exhibit sensation for the C7 component of the brachial plexus? A Lateral forearm B Lateral upper arm C Medial forearm D Medial upper arm E Third Finger

The Correct Answer is: E The sensory nerve emanating from the C7 level is responsible for sensation of the third finger. The lateral forearm is supplied by C6. The lateral upper arm is supplied by C5. The medial upper arm is supplied T1 and the medial forearm is supplied by C8.

The most commonly fractured long bone in both adults and children is which of the following? A Femur B Fibula C Humerus D Radius E Tibia

The Correct Answer is: E The tibia is the most commonly fractured long bone in the body for both adults and children. The fractures are often the result of sporting activities in the young and may occur from a simple fall in the elderly - especially those with osteoporosis. Motor vehicle accidents are another common cause of tibial fractures. Open or complex tibial fractures are sometimes associated with compartment syndromes, infection and neurovascular compromise. The femur is the strongest of the long bones and generally only sustains fractures when exposed to extreme stress, such as that experienced in a motor vehicle collision or industrial accident. Fibular fractures commonly occur with a direct below to the lateral lower leg or with extreme ankle rotational forces or excessive inversion. A high percentage of ankle fractures involve the fibula, especially in older women. Humerus fractures are relatively rare in adults, but are the second most common fractures to occur at birth - behind only the clavicle in frequency. The radius is the most commonly fractured bone in the upper extremity, but still less common in frequency than the tibia. Falls on an outstretched arm are a common mechanism for the injury

A 17 year-old male notes acute onset of dyspnea, excessive non-productive coughing, "wheezing" and upper chest tightness when running. Symptoms usually resolve in a few minutes and he is able to resume running. He is prescribed albuterol with no improvement. Which of the following is the most likely diagnosis in the patient? A Angioedema B Bronchiectasis C Cystic fibrosis D GERD E Vocal Cord Dysfunction

The Correct Answer is: E Vocal cord dysfunction (E) is commonly misdiagnosed as asthma or is a comorbidity in patients with asthma. The history of acute onset and rapid disappearance of symptoms is consistent with vocal cord dysfunction, and not typical of cystic fibrosis (C), or GERD (D).

In a boxer's fracture, the presence of how much angulation of the fifth metacarpal neck would require referral for reduction as opposed to treatment with a simple ulnar gutter splint? A Less than 10 degrees B 10-19 degrees C 20-29 degrees D 30-39 degrees E Greater than 40 degrees

The Correct Answer is: E When there is greater than 40 degrees of angulation or an extensor lag (the patient cannot fully extend the affected finger) then a referral is required. Lesser degrees of angulation without an extensor lag can generally be handled conservatively with an ulnar gutter. (Sarwark, Ed., Essentials of Musculoskeletal Care, 4 th Edition, 2010)

A 16-year-old girl presents to the office complaining of a very sore throat, swollen lymph nodes, fever, and general malaise. Her examination reveals a temperature of 102.2°F, enlarged exudative tonsils, tender cervical lymphadenopathy, and borderline enlarged spleen. Rapid strep screen is negative. Which of the following laboratory findings best supports the most likely diagnosis? A decreased white blood cell count B increased monocytes on white cell differential C thrombocytosis D decreased levels of antibody to Epstein-Barr viral capsid antigen E increased atypical lymphocytes on white blood cell differential

The Correct Answer is: E With a negative rapid strep screen, the most likely explanation for this presentation is acute infectious mononucleosis. The fever, fatigue, tonsillar hypertrophy, and splenomegaly are all classic symptoms and signs. Laboratory evaluation often includes an elevated total white blood cell count with increased atypical lymphocytes on differential. Platelets may be decreased. Initially, IgM antibodies for the Epstein-Barr virus, and viral capsid antigen (VCA) levels will be elevated. Later, the IgG levels increase and IgM normalizes

A 36-year-old man presents to the emergency department with a tight bandage around his chest to help reduce pain from a chest wall injury on his right side that occurred during mixed martial arts sparring. Physical exam reveals dullness to percussion, dry crackles and diminished breath sounds over the right lower lobe. Chest x-ray shows elevation of the right hemi-diaphragm. What is the most likely diagnosis? A Atelectasis B Bronchiectasis C Pleural Effusion D Pneumothorax E Pulmonary edema

The correct answer is (A). The patient's injury places him at risk of atelectasis, pneumothorax, or other traumatic injuries. The physical exam and chest x-ray findings are classic for atelectasis (A). Pleural effusion (C) would present with fluid in the costophrenic angle on chest x-ray. Pneumothroax (D) would typically present with findings in the upper lung fields including hyperresonance to percussion. Pulmonary edema (E) would present with increased vascular markings and evidence of fluid within the alveolar space on chest x-ray.

A 64-year-old male presents with right knee pain and stiffness while walking and going up and down stairs for the past six months. He states the pain persists all day long and is relieved with rest. There is no history of trauma and his past medical history is unremarkable. An x-ray of the right knee is performed and the following is observed: What is the suspected diagnosis based on the imaging above? A Bursitis B Osteoarthritis C Torn posterior cruciate ligament D Oblique tibia fracture E Spiral femur fracture

The correct answer is (B). Osteoarthritis (OA) is the most common cause of chronic knee pain over 45 years of age and commonly seen in weight-bearing joints when walking or climbing stairs. As disease progresses, it becomes continuous and present at night. OA in the x-ray shows a narrowed joint space (white arrow) on the medial side of the knee (B); sclerosis of the bone in the medial compartment (black arrow), which is evidence of cortical thickening; and formation of osteophytes in the medial femur (white wedge). Soft tissue injuries including a PCL tear (C) and bursitis (A), would not usually be identifiable on x-ray and would need further imaging, such as a CT or MRI. No fractures, (D) and (E), are identified on this image.

Your patient is a 4-year-old boy whose father brings him to your primary care office concerned about behavioral problems. He cannot seem to sit still in school. He seems to have a great deal of difficulty focusing for any length of time. Which of the following is required in order to make the diagnosis of attention deficit/hyperactivity disorder (ADHD) according to the DSM-V TR? A He exhibit more symptoms of inattention than hyperactivity B He must be at least 10-years-old C He behaves aggressively towards others on at least some occasions D Some impairment from the symptoms is present in two or more settings E Symptoms of inattention be present on a daily basis for 3 months

The correct answer is (D). The diagnosis of ADHD requires symptoms of both hyperactivity/impulsivity and of inattention (D). It does not require more or more prominent symptoms (A) of one or the other. Aggression (C) is seldom involved, although the impulsivity may manifest as lack of concern for others. The symptoms must be present before the age of seven (B) and must have been present for at least 6 months (E).

A 15-year-old female comes into your office with multiple symptoms of an eating disorder. What is the single feature that would lead you to diagnose anorexia rather than bulimia or binge eating disorder? A Binge eating behaviors B Compensatory measures such as purging C Preoccupation with body weight D Refusal to maintain normal body weight E Strict focus on dietary intake

The correct answer is (D). The diagnosis of anorexia requires that the patient refuse to maintain a normal body weight (D). All the other behaviors mentioned (A, B, C, E) may be shared between anorexia and bulimia. People with binge eating disorder (not fully recognized in the DSM-V-TR) display binge eating without compensatory measures, resulting in obesity.

Consideration should be given to screening patients with Type 1 Diabetes Mellitus for which of the following disorders? A sarcoidosis B Sheehan's syndrome C Sjögren's Syndrome D thyroid disease

The correct answer is (D). Type 1 diabetes mellitus (T1DM) is an autoimmune disease. As such, patients have a significantly higher risk of other autoimmune diseases, including celiac and thyroid disease. Most recommendations include screening for both diseases in patients with T1DM.

A 28-year-old female presents with a gradual progression of fatigue and pallor over the last few months. Initial CBC results show a hemoglobin of 10.4 mg/dL, hematocrit of 32%, an MCV of 112 fL, and a reticulocyte count of 3%. Which of the following is the most likely pathophysiologic mechanism responsible for her anemia? A Chronic blood loss B Defective bone marrow/stem cell function C Defective DNA production D Defective hemoglobin production E Increased destruction of red blood cells

The correct answer is (E). The time course of the patient's presentation is consistent with multiple episodes of acute hemolysis. Defects in bone marrow (B) or red blood cell precursors (C and D) are refuted by the elevated reticulocyte count. Chronic blood loss (A) would have a more insidious, gradual onset and likely result in a decreased MCV.

A 23 year-old vegan presents to the clinic complaining of fatigue. Initial CBC reveals a hemoglobin of 11.1 mg/dL and an MCV of 113 fL. Which of the following best describes the cause of the abnormality pictured in the patient's peripheral smear? (note photo taken from figure 57-5- in Harrison's) A Absent or non-functioning spleen B Failure of nuclear maturation C Intravascular hemolysis D Lead intoxication E Presence of uremia

he Correct Answer is: B Macrocytes are present on the peripheral smear and result from failure of nuclear maturation commonly secondary to vitamin B 12 or folate deficiency. Howell-Jolly bodies result from the lack of removal of nuclear material due to an absent or non-functioning spleen (A). Intravascular hemolysis (C) creates schizocytes, lead intoxication (D) and thalassemia cause basophilic stippling, and uremia (E) is associated with Burr cells.


Conjuntos de estudio relacionados

Fundamentals Nursing Prep U Chapter 34 Comfort and Pain Management

View Set

Chapter 32: The Toddler and Family

View Set

Chapter 12 Reading: Formation of Traditional and E-Contracts

View Set

Entrepreneurial Small Business 5th Edition; Chapter 4

View Set

BIOL 2321 Exam 4 Practice Test Questions

View Set

Registration and Licensing Quiz 1

View Set

1. THE MIDDLE AGES: AN INTRODUCTION

View Set